Você está na página 1de 61

RAFFLES JUNIOR COLLEGE MATHEMATICS SOCIETY

FOR JC2
STUDENTS



Preliminary Examination Strategies and Tips Notes 2008
PEST NOTES
Compilation of all chapter revision and tips taught in H2 Mathematics (9740)



Developed by:
RJC Mathematics Society
2007/2008 Batch









Version 1.33
2 September 2008
An initiative by Raffles Junior College Mathematics Society 2008 RJC Maths Society
Preliminary Examination Strategies and Tips (PEST) Notes 2008 PAGE 2


PEST NOTES
2008

PREFACE

This set of Preliminary Examination Strategies and Tips (PEST) notes is a compilation of
the contributions by Raffles Junior College Mathematics Society members to help students
to tackle problems for the H2 Mathematics (9740) Preliminary examination as well as the
General Certificate of Education Advanced Level examination.

We hope to present the more difficult concepts in a manner that is easy to grasp and
understand so as to help weaker students. In addition, we will be sharing some tips as well
as ideas that you may not be aware of that you can utilize during the examination. While
this set of notes provides a general idea of how to go about preparing for the examinations,
the PEST notes is by no means exhaustive, and is intended only as a guide. Please consult
your teachers when in doubt if necessary.

The chapters are arranged in chronological order on when they were taught during lectures
for easy reference. At the same time, as this is meant as a supplement to the lecture notes
provided, the basic fundamentals will not be mentioned in here and thus do have your notes
accompanying you for reference purposes.

We hope that you will find the PEST Notes beneficial in your quest towards excellence in
the H2 Mathematics examination. So give it a try and you will gain immensely from reading
this set of notes!


With Regards,

Raffles Junior College Mathematics Society
Batch of 2007/2008



In this set of notes, we have included several pointers to aid you in your revision:



TIP
Allows you to
gain some useful
ways to solve
problems much
quicker and
simpler.


GC TIP
Gets you to use
your graphing
calculator more
effectively and
faster in problem-
solving.


EXAM
TIP
Highlighting key
things and notes
to watch out for
and present in
your solutions
during tests.


For updates and corrections to the notes, check out www.freewebs.com/pestnotes regularly.
An initiative by Raffles Junior College Mathematics Society 2008 RJC Maths Society
Preliminary Examination Strategies and Tips (PEST) Notes 2008 PAGE 3


PEST NOTES
2008

CONTENTS


Chapter 1: Functions ............................................................................................................................................. 4
Chapter 2: Graphing Techniques .......................................................................................................................... 7
Chapter 3: Inequalities ........................................................................................................................................ 11
Chapter 4: Differentiation ................................................................................................................................... 13
Chapter 5: Integration ......................................................................................................................................... 16
Chapter 6: Differential Equations ....................................................................................................................... 20
Chapter 7: Arithmetic and Geometric Progressions ......................................................................................... 22
Chapter 8: Summation of Series and Mathematical Induction ......................................................................... 24
Chapter 9: Permutations and Combinations ..................................................................................................... 27
Chapter 10: Binomial Expansion ........................................................................................................................ 30
Chapter 11: Power Series .................................................................................................................................... 33
Chapter 12: Vectors ............................................................................................................................................. 36
Chapter 13: Complex Numbers ........................................................................................................................... 42
Chapter S1: Probability ....................................................................................................................................... 48
Chapters S2 & S3: Distribution ............................................................................................................................ 50
Chapters S4 & S5: Sampling and Testing ............................................................................................................ 55
Chapter S6: Correlation and Regression ............................................................................................................ 59








An initiative by Raffles Junior College Mathematics Society 2008 RJC Maths Society
Preliminary Examination Strategies and Tips (PEST) Notes 2008 PAGE 4



PURE
MATHEMATICS

CHAPTER 1
FUNCTIONS

This is a relatively easy topic. You are required to manipulate functions and also determine
their properties where necessary.

1.1. Basic Definitions

A function is a relation which maps each element x A e to one and only one element y B e ,
where A is the domain.

o To show that a relation is a function (vertical line test):
A relation ( ) f x ,
f
x D e , is a function if and only if any vertical line, x k = ,
f
k D e , cuts the
graph of f at one and only one point.

Figure 1.1: Graph of a function Figure 1.2: Graph of a non-function
The set of inputs of a function is known as the domain of the function (denoted by D
f
). The
corresponding set of outputs is known as the range of the function (denoted by R
f
).

A function can be represented in two ways:

2
2
: 1,
( ) 1,
f x x x
f x x x
+ e
= + e



To define a function completely, both the rule and the domain must be specified.

A function can be a (i) one-to-one relation, or a (ii) many-to-one relation.




x
y
0 x
y
0
Vertical line x = k Vertical line x = k
An initiative by Raffles Junior College Mathematics Society 2008 RJC Maths Society
Preliminary Examination Strategies and Tips (PEST) Notes 2008 PAGE 5
y = f(x)
y = x
x
y
0
y = f
-1
(x)
1.2. One-one Functions

A function f is said to be one-one if no two elements in the given domain have the same
image (output).

A function is one-one (horizontal line test) graphically if and only if any horizontal line, y =
k,
f
k R e , cuts the graph of f at one and only one point.

To show that a function is not one-one, there are two ways:
Give a counter-example. For example, the function
2
( ) 1, f x x x = + e is not one-one
since (1) ( 1) 2. f f = =
Horizontal line test: To prove that f is not one-one, we need to find just one line y =
b,
f
b R e

that cuts the graph of f at more than one point.


TIP
Look out if the function f has minimum points/maximum points. If the function has
any of these, it is not one-one and a horizontal line that cuts the graph of f at more
than one point can be found.

1.3. Inverse Functions

If f is a one-one function mapping the elements in set A to the elements in set B, then the
function that maps the elements in set B back to the elements in set A is known as the
inverse function of f.

o Condition for the Existence of Inverse Function:
The inverse function of f exists if and only if f is one-
one. This is because
1
f


will be undefined as an
element in
1
f
D

can have more than one mappings to
1
f
R

.

Definition: Let f be a one-one function. Then f has an
inverse function
1
f

defined by

1
( ) ( ) f y x y f x

= = for x edomain of f.

Hence,
Domain of
1
f


= range of f and range of
1
f


= domain of f

1 1
( ) ( ) ( ) f x f x

=
The graphical relationship between a one-one function and its inverse: the graph of
1
f

can be obtained by reflecting the graph of f about the line y = x.


If
1
( ) ( ) f x f x

=

for a certain value of x, then ( ) f x x =

for that value of x.

The domain of a function can be restricted such that the function is changed from a many-
one to one-one function, and the inverse function exists.
Figure 1.3: Graphical
representation of ( ) f x and
1
( ) f x


An initiative by Raffles Junior College Mathematics Society 2008 RJC Maths Society
Preliminary Examination Strategies and Tips (PEST) Notes 2008 PAGE 6
Example 1.1: For a function
2
( ) 2 1, f x x x x k = > where k is a real number, find the
minimum value of k such that
1
f


exists, and then find
1
f

:

Solution: Since f must be one-one for
1
f


to exist, the minimum (or maximum, depending on
question) value of k would be at a turning point. (Remember for a quadratic graph, there is
one and only one turning point)

Completing the square:
2
( ) ( 1) 2 f x x = . Thus f is at a minimum when 1 0 x = . Thus for
f to be one-one, 1. k =


Now to find the inverse:

2
2
2
Let 2 1
( 1) 2
( 1) 2
2 1 (change the subject to )
y x x
y x
x y
x y x
=
=
= +
= + +


Since 1 x > , 2 1 x y = + + (by using the domain of f to determine value of x). Hence
1
( ) 2 1, 2 f x x x

= + + > , noting that the domain of inverse function must be specified.



1.4. Composite Functions

If f is a function from set A to set B and g is a function
from set B to set C, the composite function gf is a function
from set A to set C.

Condition for the existence of a composite function:
For a composite function gf to exist,
f g
R D _
Definition: Let f and g be two functions such that
f g
R D _ , then the composite function gf is defined by ( ) ( ( )) gf x g f x = for all
f
x D e
Given two functions f and g such that

f g
R D _ ,
o
gf f
D D =
o
gf g
R R _ . In particular, if
f g
R D = , then
gf g
R R =







2
3
4

3
4
5
6

9
16
25
36

f g
A B C
gf
An initiative by Raffles Junior College Mathematics Society 2008 RJC Maths Society
Preliminary Examination Strategies and Tips (PEST) Notes 2008 PAGE 7



PURE
MATHEMATICS

CHAPTER 2
GRAPHING TECHNIQUES

This is a topic that tests your ability to transform graphs in various forms. In addition, you
will be acquainted with the various equations for conic sections and likewise perform
transformations on them.

2.1. Transformation of Graphs

There are three types of transformation for graph of ( ), 0 y f x a = >
Translating a units in (positive/negative) (x-direction/y-direction)
Stretching parallel to (x-axis/y-axis) by factor a
Reflecting in (x-axis/y-axis)

Most transformation questions would give you a graph of ( ) y f x = (or in some cases more
difficult ones like (2 ) y f x = ) and ask you to draw a corresponding graph of (2 1) y f x = + ,
(2 3) y f x =

and so on. Manipulate the variables x and y algebraically such that you reach
from the starting state to the desired state, and apply the transformations to your graph.

As a rule of thumb, translate the graph first before stretching it/reflecting it when changing
the contents inside the brackets of the function f( ) :

Imagine if you were asked to draw the graph of (2 3) y f x =

from graph of ( ) y f x = .



Due to coefficient of x being 2, method actually translates 2 3 6 =

units in the positive x-
direction instead of the supposed 3 units.

For modifications on f( ) as a whole, modify the coefficient of f( ) first (by
stretching/reflecting y) before translating (by translating y).
y = f(x)
Replace x by x 3
y = f(x 3)
Translating 3 units in
the positive x-direction
Replace x by 2x
y = f(2x 3)
Stretching parallel to
the x-axis by factor
1
2

Replace x by 2x
y = f(2x)
Replace x by x 3
Translating 3 units in the
positive x-direction (not true)

y = f(2(x 3))
= f(2x - 6)
METHOD 2
Stretching parallel to
the x-axis by factor
1
2


y = f(x)
METHOD 1
An initiative by Raffles Junior College Mathematics Society 2008 RJC Maths Society
Preliminary Examination Strategies and Tips (PEST) Notes 2008 PAGE 8
Always take note of x-intercepts, y-intercepts, asymptotes, coordinates of turning
points, and coordinates of given points (e.g. A, B, C) when transforming a graph.
Some transformation questions ask for describing the transformation, so exact
phrasing must be adopted, for example, it is stretching parallel to the x-axis by a
factor a, not stretching parallel to the x-axis with a factor a

Example 2.1: Describe the transformation of (2 ) y f x = to 3 (2 3) y f x =
.


Solution:
Step 1: Replace x by x (reflecting about y-axis): (2 ( )) (2 ) y f x f x = = +
Step 2: Replace x by (x 5) (translating 5 units in the positive x-direction):
(2 ( 5)) y f x = +

( 3) f x =
Step 3: Replace x by 2x (stretching parallel to the x-axis by factor ): (2 3) y f x =
Step 4: Replace y by -y (reflecting about x-axis): (2 3) (2 3) y f x y f x = =


Step 5: Replace y by 3 y (translating 3 units in the positive y-direction):
3 (2 3) y f x =
Step 6: Rearranging, 3 (2 3) y f x =

2.2. Modulus Functions

Refer to lecture notes for the exact steps to transform graphs such as ( ) y f x = and
( ) y f x =

from ( ) y f x = .

To check if it is correct, graph of ( ) y f x =

must be above the x-axis, i.e. all points on the
graph are non-negative; graph of ( ) y f x = is symmetrical about the y-axis.

Be careful when transforming a graph of ( ) y f x =

to the form ( ) y f x =
When trying to transform ( ) y f x =

to ( 3) y f x = + , first replace x by |x| to
transform ( ) y f x =

to ( ) y f x = , then replace x by 3 x +

to get ( 3) y f x = +
.

When trying to transform ( ) y f x =

to ( ) y f x =
,
first replace x by -x to transform

( ) y f x =

to ( ) y f x = , then replace x by x

to get ( ) y f x = .

2.3. Graphs of Rational Functions

o Sketching graphs of the form
2
, 0
ax bx c
y d
dx e
+ +
= =
+
,

numerator and denominators have no common factors
Step 1: Apply long division on
2
ax bx c
dx e
+ +
+
to convert
it to form
r
px q
dx e
+ +
+
.
Step 2: Draw the vertical asymptote
e
x
d
= and
horizontal or oblique asymptote y px q = + . With the
asymptotes drawn, the graph is divided into 4 regions,
as shown by regions A, B, C, D in Figure 2.1.
x
y
A
B C
D
Figure 2.1: Asymptotes for
graph
2
, 0
ax bx c
y d
dx e
+ +
= =
+

An initiative by Raffles Junior College Mathematics Society 2008 RJC Maths Society
Preliminary Examination Strategies and Tips (PEST) Notes 2008 PAGE 9
Step 3: Calculate the x-intercepts and y-intercepts of the graph and asymptotes
Step 4: Plot the graph using the information obtained above.
Step 5: Use the graphing calculator to check if the graph is correct.


TIP
The graph would either occupy regions B and D or regions A and C, and you can
check by substituting in a value, i.e. 1 x = , to see which region that point falls into.
The graph approaches the horizontal asymptote nearing the left and right ends of
the graph, and approaches the vertical asymptote nearing the top and bottom ends
of the graph.

o Given the graph, obtain the values of , , , , a b c d e
You can find out using the formula for vertical asymptote
e
x
d
=

and horizontal or oblique
asymptote y px q = + .

2.4. Graphs of Conic Sections

Most questions involve sketching the graph given any of the following equations:
Geometric Shape General Equation Useful values
Circle

2 2 2
x y r + = Radius of r
Ellipse
2 2
2 2
1
x y
a b
+ = for
, 0 a b >
Distance of a at leftmost, rightmost points
with respect to the centre

Distance of b at topmost, bottommost
points with respect to centre
Parabola
2
y kx =

or
2
x ky = Extent of curvature determined by k
Hyperbola
2 2
2 2
1
x y
a b
=

or
2 2
2 2
1
y x
b a
=

for
, 0 a b >
Oblique asymptotes:
b
y x
a
=

The equations given above are the cases where the curve is centered at (0, 0). For a conic
section curve centered at ( , ) m n , replace x with x m

and by replace y with y n .








An initiative by Raffles Junior College Mathematics Society 2008 RJC Maths Society
Preliminary Examination Strategies and Tips (PEST) Notes 2008 PAGE 10
Below is a flow chart to determine the type of conic section given an equation.

Step 1: How to tell if a given equation represents a circle, ellipse, parabola or
hyperbola:















Step 2: What to show for graph:
o Coordinates of the centre of the curve
o Coordinates of the four points at the top/bottom/left/rightmost part of the curve
o Radius (only for circle)
o Coordinates of the turning points
o x and y intercepts, and asymptotes (if any)
Step 3: Use the graphing calculator to check if the graph is correct

2.5. Parametric Equations

For parametric equations, x and y are each given in one equation in terms of a parameter t,
you will then be required to sketch the graph. Here are a few tips:
Find out value of t from one equation and substitute it into the other to convert it to
Cartesian form, taking into account the restrictions on the values of x and y.
Knowing trigonometric formulas might help convert parametric equations into
Cartesian form. (e.g.
2 2
sin cos 1 x x + = )
If the graph cannot be converted to Cartesian form, use GC parametric mode to plot.


GC TIP
The graphing calculator has a limitation for parametric mode, it only plots for
specified range of values of [0, 2 ] t t = . If the question requires a different domain
for the graph, extend the range by going to Window and changing Tmin & Tmax.


Is there both an x
2
term and y
2
term?

Are the coefficients of x
2
and y
2
of
different signs?
(i.e. one positive, one negative)

Yes No
Is there either an x
2
term or a y
2
term?

Parabola
Yes
Hyperbola
Yes
Are the coefficients the
same value?

No
Circle
Yes
Ellipse
No
An initiative by Raffles Junior College Mathematics Society 2008 RJC Maths Society
Preliminary Examination Strategies and Tips (PEST) Notes 2008 PAGE 11



PURE
MATHEMATICS

CHAPTER 3
INEQUALITIES

As a continuation to Chapters 1 and 2, this topic is focused on manipulating equations
involving polynomials or trigonometric elements. It tests your ability to algebraically
manipulate equations to solve for unknowns and equations.

3.1. Manipulating Equations

Usually problems in inequalities are of two forms, (i) using of the graphing calculator (for
harder equations) and (ii) direct algebraic manipulation.

o Prohibition of Graphing Calculator to Solve Inequalities
Questions usually involve polynomials, without the use of absolute value, of up to powers of
2 or 3. Here are a few strategies:
Again, always equate to zero on the RHS, as that it is more crucial here.
Example:
3 1 3 1
0
2 2
x x
x x x x

> >
+ +

Convert LHS to the same denominator for simplification purposes
Example:
(3 ) ( 2)
0
( 2)
x x x
x x
+
>
+

Multiply both sides by the denominator; i.e. convert the denominator to numerator
while maintaining the inequality sign. Expand and factorize the numerator and
denominator to linear factors:
Example:
2
(3 ) ( 2) 2 2
0 0
( 2) ( 2)
x x x x x
x x x x
+ +
> >
+ +

2
( 2)(( 1) 1) 0 x x x + + < (Take note of the sign change!)
Eliminate factors that are always positive (in the form of squares or additions to
squares)
Example: Since
2
(( 1) 1) x + is always positive, we have ( 2) 0 x x + <

for. Solving,
2 0 x < < .
To check your answer, self-plot the graph by indicating the x-intercepts based on the
linear factors. Use the graphing calculator to plot the LHS graph to determine the
general shape of the graph (whether it slopes up or slopes down initially). Shade the
corresponding areas for the inequality.

o Non-prohibition of Graphing Calculator to Solve Inequalities
These questions usually involve absolute values or trigonometric functions somewhere and
thus make it difficult to solve algebraically. Here are a few strategies:
Always equate to zero on one side say RHS. Subtracting the value of the RHS from
the LHS. This is to prevent confusion from drawing 2 graphs and finding intersects.
Plot the LHS graph using the graphing calculator!
Transfer the graph from the graphing calculator on the answer paper in order to use
information from the graph.
An initiative by Raffles Junior College Mathematics Society 2008 RJC Maths Society
Preliminary Examination Strategies and Tips (PEST) Notes 2008 PAGE 12
Use the graphing calculator to find the x-intercepts, shade the areas corresponding to
the inequality and youre done.

3.2. Type of Problems

We hereby identify two types of problems that you will find.

o Substitution
Question will ask you to find a solution set for an initial inequality, then hence find
solutions for other similar inequalities. A few tips:
Convert the inequality sign and RHS to be the same as the original inequality
Identify the substitution for x such that the original LHS transforms to the new,
mutated LHS. Popular examples are replace x by
2
, , , ln ,
x
x x x x e et cetera
Express the solution set in terms of the new form of x.
Re-convert the new form of x (such as
2
, , , ln ,
x
x x x x e ) back to x, then edit the
solution sets accordingly. For example: 1
x
e e s s should be converted to 0 1 x s s .

o Squaring
Both LHS and RHS are encapsulated in absolute values. For example: 4 1 x x > + :
Square both sides without expansion. Note: This is only possible because both sides
are guaranteed to be more or equal to 0. This ensures that the inequality sign will
not be subject to change.
Equate the RHS to zero, such that the LHS is of the form
2 2
a b
Factorize the LHS using the famous formula:
2 2
( )( ) a b a b a b = + and solve!


TIP
Take note of any restrictions to x, as these will affect the solution set of x. For instance,
1
0
2 x
>
+
can be converted to 2 0 x + > (shifting the denominator up), which gives
2 x > . However, due to the initial inequality, 2 x =

else LHS is undefined, hence the
solution set for x is ( 2, ) .

3.3. System of Linear Equations

Questions require you to express various simultaneous equations, up to an x number of
equations for x unknowns and later solve it. With the use of the program PlySmlt2, you can
easily key in the system of equations and solve it to find the unknowns. You can also key
into an augmented matrix and find the reduced row echelon form (RREF) of the matrix.

If you are using the matrix form to solve, remember a few pointers:
If the last row of the RREF consists of all zeros except the last entry which is a non-
zero entry, this means that there is no solution to the system of equations. It is
equivalent of saying
1 2
0 0 ... 0 1
k
x x x + + + =

for example, which is impossible.
Given n equations and n unknowns, and the last row of the RREF comprises all
zeros, there are infinitely many solutions and you can express all unknowns in terms
of one of them.
Note: Always find the RREF of the matrix before reaching a conclusion!
An initiative by Raffles Junior College Mathematics Society 2008 RJC Maths Society
Preliminary Examination Strategies and Tips (PEST) Notes 2008 PAGE 13



PURE
MATHEMATICS

CHAPTER 4
DIFFERENTIATION

As part of the calculus module, knowing how to differentiate will come useful in subsequent
chapters such as power series and also in differential equations. It is a powerful tool to
discover the properties of equations in the Cartesian plane and also derive useful results.

4.1. The Basics

Here are a few basic operations to note for differentiation:
o Is there a power added to an entire function?
o If yes, then
( ) ( )
1
( ) ( ) ( )
n n
d
f x n f x f x
dx

' = (note the use of Chain Rule)


o Is it a product of 2 functions?
o If yes, then ( ) ( ) ( ) ( ) ( ) ( )
d
f x g x f x g x f x g x
dx
' ' = + (Product Rule)
o Note:
( )
2
( ) ( ) f x g x may be expressed as
( ) ( )
2 2
( ) ( ) f x g x
o Is it a quotient of 2 functions?
o If yes, then
( )
2
( ) ( ) ( ) ( ) ( )
( )
( )
f x f x g x f x g x d
dx g x
g x
' '
= (Division Rule)
o Note: Be careful about the minus sign in the numerator. If you find it difficult to
remember this, substitute
1
( )
( )
h x
g x
= and perform the product rule instead on
( ) ( ) f x h x .

Other than these few basic ones, you need to know these as well:
y (in the (
d
y
dx
))
dy
dx

( ) f x
e
( )
( )
f x
e f x '
ln ( ), 0 f x x >
( )
( )
f x
f x
'

sin ( ( ))
n
A f x
1
sin ( ( )) cos( ( )) ( )
n
nA f x f x f x

'


Note: Divide the thinking process in the above
three blocks power * coefficient * original
with power of ( 1) n , then differentiate
sin( ( )) f x (not forgetting differentiating ( ) f x )
1
sin ( ( )) A f x


2
1
( )
1 ( ( ))
A f x
f x
'


(Other trigonometric or inverse trigonometric functions can be differentiated similarly and
hence not shown above)
An initiative by Raffles Junior College Mathematics Society 2008 RJC Maths Society
Preliminary Examination Strategies and Tips (PEST) Notes 2008 PAGE 14
Strictly Increasing: The segment in which
y |as x | on the graph of ( ), 0
dy
f x
dx
>
Concave Upwards: The segment in which
dy
dx
| as x | on graph of
2
2
, 0
dy d y
dx dx
>
dy
dx
,
2
2
d y
dx
>0
Strictly Decreasing: The segment in which
y +

as x | on the graph of ( ), 0
dy
f x
dx
<
Concave Downwards: The segment in which
dy
dx
+ as x | on graph of
2
2
, 0
dy d y
dx dx
<
2
2
d y
dx
>0
4.2. Other Considerations

There are also other factors to consider when doing differentiation:
Other order derivatives: differentiate from n
th
order to
th
( 1) n

order and so on, until
a trend is seen
Implicit differentiation: Differentiate every term with respect to a variable. Thus, if
differentiating things like ( ) f y with respect to x, add the notation
dy
dx

at the back.
Of course, if you come across things such as ( ) ( ) f y g y

when differentiating with respect to x,
make use of Product Rule, Division Rule or Chain Rule as highlighted above.


TIP
When both x and y variables are present in an equation to be differentiated, implicit
differentiation is recommended, as compared to expressing y in terms of x, and
then differentiate
dy
dx
. This may be very difficult and time-consuming.

Sometimes, to simplify matters, manipulation to the original term should be done so as to
make differentiation simpler and also reduce errors. For example, if differentiating
ln( ( ) ( )) f x g x , split it into ln( ( )) ln( ( )) f x g x +

as there is no need to use Product Rule.

4.3. Applications of Differentiation

Here are some basic properties of graphs with respect of calculus:
Trend of changes in the graph of the function ( ) f x

Trend of changes in the graph of the derivative
dy
dx

2
2
, 0
dy d y
dx dx
>
dy
dx
|

y
Concave
downwards
y
Concave
upwards
x
y
Strictly
decreasing
y
Strictly
increasing
x
Figure 4.1: Graphs for strictly increasing and decreasing functions
Figure 4.2: Graphs for concave upwards and downwards functions
x x
An initiative by Raffles Junior College Mathematics Society 2008 RJC Maths Society
Preliminary Examination Strategies and Tips (PEST) Notes 2008 PAGE 15
o Transforming ( ) y f x = to ( ) y f x ' =
Whenever the graph of ( ) y f x =

is horizontal or there are turning points, then ( ) 0 y f x ' = =
and hence the graph of ( ) y f x ' =

must touch or cut the x-axis.

o Transforming ( ) y f x =

to
1
( )
y
f x
=
Label these in your graph:
x-intercept of ( ) y f x =

becomes vertical asymptote of
1
( )
y
f x
=

and vice versa
If the y-intercept k = , then the y-intercept of
1
( )
y
f x
=

is
1
k

Maximum point of y becomes the minimum point of y
Note the following:
As
1
( ) , 0
( )
f x
f x
, hence as ( ) f x increases,
1
( ) f x

decreases
When
1
( ) 1, 1
( )
f x
f x
= = ; also if
1
( ) 0, 0
( )
f x
f x
> > (same sign)

o Transforming ( ) y f x =

to
2
( ) y f x =
Apply the following steps:
Remove everything below the x-axis.
Sketch graph of ( ) y f x =
Treat accordingly the various intersections/touches
of ( ) y f x = with the x-axis
o ( ) y f x =

has single distinct root: vertically
perpendicular to x-axis at x a =
o ( ) y f x =

has 2 equal roots: sharp point touching
x-axis at x a =
o ( ) y f x =

has more than 2 equal roots: as x a ,
tangent to x-axis
Reflect this graph about the x-axis to get
( ) y f x =

4.4. Rate of Change
Differentiation is useful in the real world context in various
situations. Solving questions involving such concepts can be done through the following
steps:
Step 1: Find known derivatives (with respect to time) that are given, for e.g.:
dV
dt

Step 2: Link the known derivative (dV) to the unknown, e.g.: dr in a sphere, using
the formula given, for e.g.
3
4
3
V r t =
Step 3: Find the other differential, say
dr
dt
, using chain rule:
dV dV dr
dt dr dt
= .
Step 4: Find maxima / minima using second derivative


x a = x
Tangent to x-axis

x a =
Near-vertical to x-axis

x
Figure 4.3: Graphs of
( ) y f x =

when x approaches
a root of

( ) y f x =
An initiative by Raffles Junior College Mathematics Society 2008 RJC Maths Society
Preliminary Examination Strategies and Tips (PEST) Notes 2008 PAGE 16



PURE
MATHEMATICS

CHAPTER 5
INTEGRATION

As part of the calculus module, knowing how to do integration is also important. It will come
in handy to solve differential equations as well as calculate volumes of solids of revolution that
can be applied in physics.

5.1. The Basics

You should be familiar with this list of integrals. (Remember to add the arbitrary constant c
at the back of the expression, and the absolute signs for ln x!)
( ) f x ( ) f x dx
}

x
n

1
1
n
x
c
n
+
+
+

1
x

ln x c +


x
e
x
e c +
cos x

sin x + c
sin x cos x + c
sec
2
x tan x + c


TIP
In a complicated expression, before launching into integration by parts, substitution
etc, check to see if the expressions are derivatives of one another or the derivatives
of some functions are present: e.g.
tan sec
2
1 sin

cos
x x
x
e dx
x
+
+
}
, upon checking one will
realize that
2
1 sin
tan sec
cos
d x
x x
dx x
+
+ = . The question will usually hint to this
relation by asking one to find the integral/derivative of some expression (or by the
relatively little marks allocated!)


5.2. Integrals Involving Certain Expressions

Integrals come in many forms and sizes. So it is important to know the various techniques to
integrate such expressions.
o Integrals with fractions:

( )
ln ( )
( )
f x
dx f x c
f x
'
= +
}


1
2 2
1 1
tan
x
dx c
a a a x

| |
= +
|
+
\ .
}
(from MF15)
An initiative by Raffles Junior College Mathematics Society 2008 RJC Maths Society
Preliminary Examination Strategies and Tips (PEST) Notes 2008 PAGE 17
Use partial fractions where necessary to simplify matters
Sometimes when you think it is necessary to use integration by parts, e.g.
2
x
dx
x +
}
,
it is possible to just convert it to a proper fraction, e.g.
2 2 2
1 - 2ln 2
2 2 2
x
dx dx x x c
x x x
+
= = + +
+ + +
} }

If 0 B = , use tan
-1
to integrate
2
C
cx d +

Use MF15 to figure out the appropriate expression to integrate

o Trigonometric formulae:
Where necessary use identities to simplify expressions to get expressions that can be
integrated:
sin
2
x + cos
2
x = 1
Dividing by cos
2
x : tan
2
x + 1 = sec
2
x
Dividing by sin
2
x : cot
2
x + 1 = cosec
2
x
If given sin
2
x or cos
2
x, use cos2x = 2cos
2
x 1 = 1 2sin
2
x

o Substitution:
Where necessary, make the substitution ( ) x g u = . Then
2 2
1 1
( ) ( )
x u
x u
dx
f x dx F u du
du
=
} }

where ( ) F u is ( ) f x after substitution of x to u. (using ( ) x g u = )
Remember to replace the lower and upper limits by
1
( ) u g x

=
Convert ( ) f x into ( ) F u then multiply by
dx
du
in order to convert all terms with
respect to u

o Integration by parts:
Use
dv du
u dx uv v dx
dx dx
=
} }
where LHS is expression you want to integrate
Use this method when integrand is a product of 2 distinct functions
How to choose u: LIATE (log, inverse trigonometric, algebraic, trigonometric,
exponential) in order of preference
When you see a function that appears with its derivative, set it as
dv
dx
: e.g. in
( ) ( ) ( ) f x f x g x dx '
}
, set ( ), ( ) u g x v f x = = as ( ) ( )
dv
f x f x
dx
' =
If final expression includes an integral of the form
dv
k u dx
dx
=
}
, we dont have to do
integration by parts anymore. Just algebraically manipulate the expression to solve.
For example, sin2 sin2 2 cos2 4 sin2
x x x x
e x dx e x e x e x dx =
} }
. By algebraic
manipulation,
sin2 2 cos 2
sin2
5
x x
x
e x e x
e x dx

=
}

An initiative by Raffles Junior College Mathematics Society 2008 RJC Maths Society
Preliminary Examination Strategies and Tips (PEST) Notes 2008 PAGE 18
Below is a flow chart depicting how to identify the types of integration questions:





















5.3. Applications of Integration

o Areas
The definite integral:
0

lim
x b
x b
x a
x
x a
y x y dx
o
o
=
=
=

=
=

}

Summing up rectangles of area y x o under a curve
Does the integrand look like the form ( ) ( ) kf x f x ' ?

Integrate directly to get
( ) k f x c + .
Does the integrand involve trigonometric
functions?

Apply trigonometric identities to simplify. If it
cannot be solved, try integration by parts
See if it can be expressed as partial fractions, or integrated directly using MF15
or using conversion to proper fraction.
Example of using MF15:
1
2
2 2
1 1 2 1 2
sin
2 2
3 3 4
( 3) (2 )
x
dx dx c
x
x

| |
= = +
|
\ .

} }

If it cannot be solved, try integration by parts.
Does the integrand involve fractions?

Does the integrand involve f(x) that can be expressed as a polynomial, and a function
that involves as a trigonometric, exponential or logarithmic function?

Use integration by parts, choosing u and
dv
dx
according to the LIATE rule.
Example: ln x x dx
}
, choose ln ,
dv
u x x
dx
= =
Yes No
Yes
No
Yes
No
Yes
a
b
y
x 0
y = f(x)
Figure 5.1: Area under the
graph y = f(x)
An initiative by Raffles Junior College Mathematics Society 2008 RJC Maths Society
Preliminary Examination Strategies and Tips (PEST) Notes 2008 PAGE 19
y-axis: ( )
y d
y c
g y dy
=
=
}
= area bounded by curve x = g(y), y-axis and y = c, y = d
Using GC to evaluate area under a curve: First use GC to find x-intercepts (limits of
integral)
Area between curves y = f(x) and y = g(x) ( ( ) ( ) f x g x > within
interval) = ( ( ) ( ))
b
a
f x g x dx
}
. This applies even if the y values
of the curves are negative.
Useful tip: If the graph of y = f(x) passes through the x-axis,
then to find
x b
x a
y dx
=
=
}
we need not split up into positive and
negative regions of the graph, we can just use
x b
x a
y dx
=
=
}
. Similarly, if the graph of x
= g(y) passes through the y-axis, then we can just use ( )
y d
y c
g y dy
=
=
}
.
Parametric equations : ( )
t q b
a t p
dx
y dx y dt
dt
=
=
=
} }
(dont forget to convert limits of integral)

o Volumes of Revolution
Rotate region bounded by curve and x-axis 2t rad about x-axis:
Volume =
2

x b
x a
y dx t
=
=
}

Rotate region bounded by curve and y-axis 2t rad about y-axis :
Volume =
2

y c
y d
x dy t
=
=
}

If graph is symmetrical about x-axis, we obtain volume of solid by rotating t rad
about the x-axis
Volume of solid when a region between two curves is rotated = Volume of solid when
C
1
is revolved Volume of solid when C
2
is revolved (
1 2
C C > within interval)




Figure 5.3: Area between two curves C
1
and C
2


c
x 0
d
x = f(y)
d
c
x 0
C
1
C
2
a b
x
0
C
2
y
y
y
C
1
Figure 5.2: Area under
the graph x = f(y)
An initiative by Raffles Junior College Mathematics Society 2008 RJC Maths Society
Preliminary Examination Strategies and Tips (PEST) Notes 2008 PAGE 20



PURE
MATHEMATICS

CHAPTER 6
DIFFERENTIAL EQUATIONS

In a continuation to the calculus module, many physical phenomena or situations can be
modeled using differential equations. It is important that you have learnt the prerequisites in
Chapters 4 and 5 so that you can easily manipulate the equations to find the unknowns.

6.1. Solving Differential Equations

Differential equations are often of form ( )
dy
f x
dx
= . The important part of this topic will be
to solve them, that is, to find the solutions that make the equation hold true and also fit in
with possible measurements or data collected.

Below is a step-by-step guide on how to solve for y given
dy
dx
:
Is equation of the form ( )
dy
f x
dx
= ?
o If yes, then integrate directly: ( ) ( )
dy
dx f x dx y f x dx
dx
= =
} } }
(see Chapter 5
for more details on how to integrate various functions f(x).)
Is equation of the form ( )
dy
f y
dx
= ?
o If yes, then transform it and integrate both sides with respect to x:
1 1 1
( ) 1 1
( ) ( ) ( )
dy dy dy
f y dx dx dx
dx f y dx f y dx f y
= = = =
} } }

Is there substitution involved, e.g. presence of another variable v?
o If yes, then substitute completely by eliminating all
dy
dx
and y. The substitution
will generally involve y expressed in x and another variable (for example v)
o Find
dy
dx
from the substitution equation. You should get something with
dv
dx
on
the other side, since v is a variable (Chain Rule).
o Substitute for y and
dy
dx
you just got into the original equation
o Manipulate to get v and
dv
dx
on one side, x on the other: ( )
dv
f v k
dx
= , where k is
some constant and solve it with ( ) f v dv k dx =
} }
.
o Substitute y = kv or other relevant substitutions done earlier on to get an
equation with y as the subject and not v!


TIP
If there is
2
2
d y
dx
, apply the above steps, but remember to integrate twice. There will
be 2 constants, in the form ( ) Ax B + , with A being the constant term derived from
the 1
st
integration and B from the 2
nd
.
An initiative by Raffles Junior College Mathematics Society 2008 RJC Maths Society
Preliminary Examination Strategies and Tips (PEST) Notes 2008 PAGE 21
6.2. Family of Solution Curves

After solving for the solutions of a differential equation, you may be required to sketch the
graph of ( ) y f x c = + to demonstrate the shape of the solution curves. Here are a few simple
pointers to note:
Generally sketch 3 cases, with 0, 0, 0 c c c = > < .
Use simple values for c, like 1 or 1.
Do check carefully if there is any restriction on the domain/range. For example, the
question states/implies that time, t, of volume V cannot be less than 0.

6.3. Forming and Solving Differential Equations

Forming such equations is usually relatively straightforward given the details in the question.
There are always two catchwords to look out for which will tell you how to construct the
differential equation given the variables mentioned: rate and proportional.
Rate:
dx
dt
, where x is the property whose rate of change is being measured.
Proportional: if we say y is proportional to x, then y = kx, where k is an arbitrary
constant.
o Hence when the question says the rate is proportional to a variable, say a , it
means that ( )
dx
k g a
dt
= where k is a constant and ( ) g a is some function of a.
o Likewise, when rate is inversely proportional to some variable a, it means that
( )
dx k
dt g a
= .

If necessary, substitute in values or given data for variables in the equation determined
above in order to find out constants such as k and c. This will help refine your solution to
the differential equations.


EXAM
TIP
Usually the variables mentioned are given in the question, but if necessary, declare
the variable and constants (e.g. let x be the length of the cylinder) yourself.



TIP
Practice makes perfect. Remember to practise your integration so that you can
easily manipulate differential equations around to find the solutions. In addition, if
the expression, after integrating, is very complicated, do not panic, as sometimes it
is very difficult to simplify. Differentiate what you have to check whether you get
the correct answer.






An initiative by Raffles Junior College Mathematics Society 2008 RJC Maths Society
Preliminary Examination Strategies and Tips (PEST) Notes 2008 PAGE 22





PURE
MATHEMATICS

CHAPTER 7
ARITHMETIC AND GEOMETRIC
PROGRESSIONS

In this chapter, pattern-finding and deriving a general formula for terms or sum of an
arithmetic series and geometric series are important. Manipulation skills are essential in
doing well for this topic.

7.1. Knowing The Basics

o Solving Questions Involving Arithmetic Progression (AP)
When you encounter a question involving an AP, do the following things:
List out the following (wherever possible)
o First term, a
o Common difference, d
o The various u
k
where k
+
e given (e.g.
11
354 u = )
o The various S
k
where k
+
e given (e.g.
50
2008 S = )
o No. of terms, n
Apply one or more of the following formulae to find out the unknown(s)
o ( 1)
n
u a n d = +
o (2 ( 1) ) ( )
2 2
n n
n n
S a n d a u = + = +
o Note that it may be necessary to use the same formula more than once, especially
when more than one u
k
or S
k
is given.
If the question is asking for the greatest/least n such that , form an inequality
using the above formulae and solve accordingly.
If the question asks you to prove that the terms of a series form an arithmetic
progression, find
n
u and
1 n
u

(they can be expressed in terms of n) and show that


1 n n
u u k

= for all n, where k is a constant. If the formula for


n
S is given, use the
formula
1 n n n
u S S

=

and then solve for all n

o Solving Questions Involving Geometric Progression (GP)
When you encounter a question involving a GP, do the following things:
List out the following (wherever possible)
o First term, a
o Common ratio, r
o The various u
k
where k
+
e given (e.g.
11
800 u = )
o The various S
k
where k
+
e given (e.g.
50
9042 S = ) or sum to infinity S


o No. of terms, n
Apply one or more of the following formulae to find out the unknown(s)
o
1 n
n
u ar

=
o
1
1
n
n
r
S a
r

or , 1
1
a
S r
r

= <


An initiative by Raffles Junior College Mathematics Society 2008 RJC Maths Society
Preliminary Examination Strategies and Tips (PEST) Notes 2008 PAGE 23
o Note that it may be necessary to use the same formula more than once, especially
when more than one u
k
or S
k
is given.
If the question is asking for the greatest/least n such that , form an inequality
using the above formulae and solve accordingly.

When you encounter a question that involves both AP and GP and/or more than one
AP/GP, do not fret! Just segment your paper into 2 or 3 columns, and apply the relevant
steps above for each AP or GP. Be systematic!

Example 7.1: There exists an arithmetic progression with an integer common difference d
such that some five consecutive terms
1 2 5
, , , u u u

add up to 35, and their product is 3640.
Find the first term
1
u .

Solution: Considering the third term
3
u , we have
1 2 5 3 3 3
( 2 ) ( ) u u u u d u d u + + + = + + +

3 3 3 3
( ) ( 2 ) 5 35 7 u d u d u u + + + + = = = . Then
1 2 5
(7 2 )(7 )7(7 )(7 2 ) u u u d d d d = + + and
you can easily solve for d either using a graphing calculator or using the fact that
4 2 2 2
7(7 2 )(7 )(7 )(7 2 ) 3640 4 245 1881 (4 209)( 9) 0 d d d d d d d d + + = + = = . Solving,
we have 3, 52.25 d = (rejected). Thus
1 3
2 7 2(3) 1,13 u u d = = = .





























An initiative by Raffles Junior College Mathematics Society 2008 RJC Maths Society
Preliminary Examination Strategies and Tips (PEST) Notes 2008 PAGE 24





PURE
MATHEMATICS

CHAPTER 8
SUMMATION OF SERIES AND
MATHEMATICAL INDUCTION

In this chapter, just like in Chapter 7, you will need good algebraic and manipulation skills to
tackle summation questions and sometime simplify several such summations. In addition, for
mathematical induction, you may have to use your skills, through pattern-gazing, to determine
the formula of a series or recurrence and later prove it.

8.1. Knowing The Basics

Whenever you encounter a summation question, do not ever forget the following properties
and basics so as to avoid creating unnecessary obstacles and of course undue stress for
yourself!
Apply the relevant formulae and work out the solution carefully!
List of important concepts/formulae
o
1 n n n
u S S

= (from Chapter 7)
o Taking out constants to simplify calculations: ( ) ( )
n n
r m r m
af r a f r
= =
=

where a is
a constant
Note that the number of terms in the above is 1 n m +
o Separating the terms (so that you can apply any formula if possible where
needed):
( )
( ) ( ) ( ) ( )
n n n
r m r m r m
af r bg r a f r b g r
= = =
+ = +

where a and b are
constants
o Summation of constants:

( 1)
n
r m
a n m a
=
= +


o Look out for APs and GPs and apply their summation formulae

2
1
( 1)(2 1)
6
n
i
n n n
i
=
+ +
=



2
3
1
( 1)
2
n
i
n n
i
=
| | +
=
|
\ .






Example 8.1: Given
2 2
2 1
( ) ( 1)
( 1)
r
f r f r
r r
+
+ =
+
, determine
2 2 2 2 2 2
5 7 2 1
2 (3) 3 (4) ( 1)
n
n n
+
+ + +
+

.


Solution: Notice that
2 2
2 2 2 2 2 2
2 1 2 1 1 1
( ) ( 1)
( 1) ( 1) ( 1)
r r r r
f r f r
r r r r r r
+ + +
+ = = =
+ + +
. Thus
2
1
( ) f r
r
= .
Note that it is necessary for the
summation to start from 1 : r =
If it does not start from 1, apply
1
1 1
( ) ( ) ( )
n n k
i k i i
f x f x f x

= = =
=



An initiative by Raffles Junior College Mathematics Society 2008 RJC Maths Society
Preliminary Examination Strategies and Tips (PEST) Notes 2008 PAGE 25
Hence
2 2 2 2 2 2 2 2
2
5 7 2 1 2 1
2 (3) 3 (4) ( 1) ( 1)
n
r
n r
n n r r
=
+ +
+ + + =
+ +


( )
2
( ) ( 1)
(2) (3)
(3) (4)
(4) (5)

( ) ( 1)
n
r
f r f r
f f
f f
f f
f n f n
=
= +
=
+
+
+


(2) ( 1) f f n = +

2
1 1
4 ( 1) n
=
+


8.2. Mathematical Induction

Mathematical induction at H2 level is relatively straightforward. Here is a general guide on
how to approach such questions:
Understand the concept:
o First you show that the base case
1
P is true (by direct computation)
o Then you assume that
k
P is true for some k
+
e . You want to prove that
1 k
P
+
is
true whenever
k
P is true.
o Domino effect: Since the base case is true, subsequent cases are also true.
Write in a proper format: It is important to present it clearly so that the induction
and proof is complete and explicit:
o Let P
n
be the statement ..
When 1 n = , LHS =
RHS = ...
Since LHS RHS = ,
1
P is true.

Assume
k
P is true for some k
+
e (or whatever range question states):
i.e.

To prove
1 k
P
+
is true: i.e.

Now, ... LHS =
RHS =



i.e.
1 k
P
+
is true whenever
k
P is true.

Since
1
P is true, by Mathematical Induction,
n
P is true for all n
+
e (or
whatever range question states)


An initiative by Raffles Junior College Mathematics Society 2008 RJC Maths Society
Preliminary Examination Strategies and Tips (PEST) Notes 2008 PAGE 26
Solving Induction Questions
There are two types of techniques to tackle questions on induction, especially if certain
conditions are posed. They are namely (i) summation-type and (ii) recurrence-type.

For (i) summation-type questions, after proving the base case, you will need to prove the
general case. To do so, using the assumption, add the relevant
th
( 1) n + term on both sides of
the equation, then manipulate the RHS (with the general formula in terms of n) and simplify
to convert the RHS in terms of ( 1) n + and prove the case.

For (ii) recurrence-type questions, after postulating the formula for the term
n
u in terms of
n, use the recurrence given to find
1 n
u
+
and of course write it in terms of ( 1) n + . Then you
are done.



































An initiative by Raffles Junior College Mathematics Society 2008 RJC Maths Society
Preliminary Examination Strategies and Tips (PEST) Notes 2008 PAGE 27






STATISTICS

CHAPTER 9
PERMUTATIONS AND
COMBINATIONS

Many types of questions can appear in this section. You will need good counting and
manipulation skills to consider various cases in a scenario and thus deduce the number of
possibilities available. It is understandably hard to count cases in certain scenarios and it
takes practice, different perspectives and perhaps a touch of inspiration to tackle questions. In
addition, the concept of probability (see Chapter S1) can go concomitantly with the concepts
in this chapter.

9.1. Basic Counting Principles

Permutations and combinations (P&C) often involve the four basic operators. However, it is
easy to get confused on when to use addition (or subtraction) and multiplication (or
division). Here is a distinguishing factor on how to decide on such matters:
If k events can occur, each with
i
a ways to do so, then the number of ways such that
only one (not two or more) of these events can occur is
1 2 i k
a a a a = + + +

.
(Addition Principle)
If an event A is decomposed into smaller ordered events such that there are k events
and each of the events has
i
a ways to do so, then the number of ways in which event
A can occur is
1 2 i k
a a a a =
[
. (Multiplication Principle)

Example 9.1: There are 5 dogs and 3 cats in a pet shop. Find the number of ways to buy (i)
one pet, (ii) 1 cat and 1 dog.

Solution: (i) As one buys a dog in place of a cat and vice versa, number of ways to buy a pet
= number of ways to buy a dog + number of ways to buy a cat = 5 3 8. + =
(ii) Number of ways to buy 1 cat and 1 dog = number of ways to buy a dog + number of
ways to buy a cat = 5 3 15. =

9.2. Choosing and Permuting

Another confusing concept that surface in P&C is the idea of when to choose and when to
permute. Given the myriad of questions and scenarios that pose various conditions, it can be
hard at times to decipher or pinpoint the exact methods to use. On the next page is a flow
chart to categorize the various formulae one should use when encountering such a problem.






An initiative by Raffles Junior College Mathematics Society 2008 RJC Maths Society
Preliminary Examination Strategies and Tips (PEST) Notes 2008 PAGE 28

















No





























Is the order of objects important?
Key Words (not exhaustive):
Important: arrange / order /sit in a row or around a table
Not Important: choose / select / divide into groups / form committees
Can each object be repeated any number of times?
The number of ways
to obtain an
arrangement of r
objects (taking these
objects from a set of
n distinct objects) is:
n
r

Example: Number of
ways of forming any
word using four
letters from the
English alphabet
(e.g. dfss): n = 26,
r = 4. No. of ways =
26
4
= 456976
Are all the objects distinct?
No
The number of ways to
choose r objects from a
set of n distinct objects is:

!
!( )!
n
r
n
C
r n r
=


Example: Choosing 2 CT
reps from a class of 28
people: n = 28, r = 2.
No. of ways
28
2
378 C = =
Yes No
Yes
Yes
No
No
The number of ways to
obtain an arrangement
of r objects (taking
these objects from a
set of n distinct
objects) is:
!
( )!
n
r
n
P
n r
=


Example: Arranging 5
people to sit in a row:
n = 5, r = 5. No. of
ways
5
5
120 P =
(Note 0! 1 = )
The number of ways to arrange n
objects where:
n
1
are of Type 1,
n
2
are of Type 2,
n
k
are of Type k is:
1 2
!
! ! !
k
n
n n n

(Note
1 2 k
n n n n + + = )
Example: Arrange 2 red balls, 3
green balls and 5 blue balls in a
row: n = 2 + 3 + 5 = 10, n
1
= 2,
n
2
= 3, n
3
= 5. No. of ways =
10!
2520
2! 3! 5!
=
An initiative by Raffles Junior College Mathematics Society 2008 RJC Maths Society
Preliminary Examination Strategies and Tips (PEST) Notes 2008 PAGE 29
9.3. Other Considerations

In addition on which one of the formulae to use in the table above, there are also a few more
things to note to tackle questions with other conditions imposed.
Circular Permutations:
o To arrange q objects in a circle: divide the total number of arrangements in a row
by q. An important assumption is that the positions around the circle are non-
distinct, that is, the positions around the circle of each item placed do not matter.
Group similar and adjacent objects together.
o If two or more similar objects must be adjacent, treat them as a single group.
Then arrange, if necessary, the remaining entities. Thus if there are n objects and
some k of them are grouped together, with n k > , the number of groups left
1 n k = + . Number of ways to arrange the groups in a row ( 1)! n k = +
(Remember to consider the number of arrangements of these objects within the
group!)
Consider different cases using the Addition Principle
o If its impossible to count in a direct way, split the given situation into separate
non-overlapping cases and count the number of arrangements/selections on a
case-by-case basis. (Make sure each arrangement/selection can only fulfill one
case and not more, else you will be double-counting)
Count in a different manner
o If there are too many cases to count, after you split a situation into separate
cases, it is necessary to count in a different way to save time and also reduce
possibilities of computational errors.
o Use the complement method if necessary. For example, if the number of ways to
achieve event A with no restrictions is x, and the number of ways to achieve A
possessing factor Q is y, then y = x number of ways to achieve A without factor
Q. This is also useful in the chapter on probability. (See Chapter S1)


TIP
In a P&C question, you do not have to count everything at the same time! Develop
a systematic, step-by-step approach on how to arrange the objects without double-
counting or missing out cases. As long as the arrangements are in succession, you
can count the number of arrangements part-by-part without losing track of the
restrictions imposed. It is better to arrange the variables with no restrictions first
as part of this approach.











An initiative by Raffles Junior College Mathematics Society 2008 RJC Maths Society
Preliminary Examination Strategies and Tips (PEST) Notes 2008 PAGE 30



PURE
MATHEMATICS

CHAPTER 10
BINOMIAL EXPANSION

This is a relatively easy topic. Questions are straightforward and you are required to
manipulate the formulae (provided in MF15) to solve some basic questions.

10.1. Binomial Expansion for Positive Integral Index

Here are some basic facts to note:

0 1 1 2 2 0
0
( ) ...
0 1 2
n
n n n n n n i i
i
n n n n n
a b a b a b a b a b a b
n i

=
| | | | | | | | | |
+ = + + + + = | | | | |
| | | | |
\ . \ . \ . \ . \ .

, where:

!
!( )!
n
r
n
n
C
r
r n r
| |
= = |
|

\ .
, 0 r n s s
A good way to remember the above equation is that the power of a and the power of
b sum up to n at all times, and the power of b is the value of r in
n
r
| |
|
\ .

The
th
( 1) r + term in the above equation is
n r r
n
r
a b
| |

|
|
\ .
.

Questions involving binomial expansion for positive integral exponents can be classified into
two types. These questions undoubtedly involve the use of the above formula.

o Expansion and simplification of ( )
n
a b +
There are two tips to expand: (i) always look out for the directions of the question (such as
expand in increasing powers of x and expand up to the first r terms), and (ii) expand with
only two terms present. For the latter, this means that common terms must be combined
first and after binomial expansion is performed, simplification of these terms should be done.

Example 10.1: By considering the expansion of
8
(1 ) y + , find the first three terms in the
expansion, in ascending powers of x, of
2 8
(1 2 3 ) x x + + .

Solution: Using
2
2 3 y x x = + ,
8 8 2 8 8 2 7 2 2 6
1 2
(1 2 3 ) 1 (2 3 )1 (2 3 ) 1 x x x x x x
| | | |
| |
| |
\ . \ .
+ + = + + + + +.
Expanding and simplifying,
2 8 2
(1 2 3 ) 1 16 136 x x x x + + = + + .

o Finding the coefficient of the r
th
term or the independent term
There are no tricks except using the formula
1 ( 1)
1
r n r
n
r
a b
| |

|
|
\ .
for the r
th
term. Occasionally, you
will be asked, for some expansions, of the term independent of a certain variable.

Example 10.2: Find the term independent of x in the expansion of
3
2 8
( )
x
x + .

Solution: The
th
( 1) r + term in the expansion is
3
8 8 8 8 8 3 8 4 2
2 2
r
r r r r r r
r r r
x
x x x
| | | | | |
| |

| | |
|
| | | |
\ .
\ . \ . \ .
= = . In order
for the term to be independent of x, the power of 0 x = , thus 2 r = . Hence the term
independent of x is
8 2 8 8
2
2 112 x
| |

|
|
\ .
= .
An initiative by Raffles Junior College Mathematics Society 2008 RJC Maths Society
Preliminary Examination Strategies and Tips (PEST) Notes 2008 PAGE 31
10.2. Binomial Expansion for Rational Index

Here are more basic facts to note:

( 1) ( 1)...( 1)
(1 ) 1 ... ...
2! !
n r
n n n n n r
x nx x x
r
+
+ = + + + + + . There are infinitely
many terms in this expansion. As a result, questions will often direct you to expand
up to a certain r
th
term in increasing powers of x, such as up to x
2
or x
3
.
Range of validity of this expansion is when 1 x < (Important fact)
- n is usually either a negative integer or a fraction. If n is a positive integer, then all
terms starting with the
th
( 2) n + term
2
( 1)...( ( 1) 1)
0
( 2)!
n
n n n n
x
n
+
+ +
= =
+
. This gives
us back the formula mentioned in section 10.1.
- Know the shortcut expansions:
1 2 3
1 2 3
2 2 3
(1 ) 1
(1 ) 1
(1 ) 1 2 3 4
x x x x
x x x x
x x x x

= + + + +
+ = + +
= + + + +




TIP
It is very important to manipulate your binomial expansion first such that you have
a 1 as one of the terms! Take out constant terms (without forgetting the powers)
to do so. For example, manipulating
3
(2 3 ) x

+ gives
3 3 3
3 1 3
2 8 2
2 (1 ) (1 ) x x

+ = +
in which we can apply the formula. In addition, do not forget to include the range
of validity for x. In this case,
3 3
2 2
1 x x = < which gives
2
3
x < .

Example 10.3: Expand
1 3
(1 ) (2 ) x x

in ascending powers of x as far as the term in x
2
.

Solution:
Step 1: Make all terms to the form (1 ( ))
n
f x + :
3 3
1 3 1 1 3
(1 ) (2 ) (1 ) 2 1 (1 ) 2 1
2 2
x x
x x x x


| | | | | |
= =
| | |
|
\ . \ . \ .

Step 2: Utilize any shortcut expansions / expand out the whole term to the term
specified (in this case x
2
)
3 2
1 3 2 3
( 3)( 4) 3
(1 ) 2 1 (1 ...) 2 1 ...
2 1 2 2! 2
x x x
x x x


| | | |
| | | | | |
| |
= + + + + + +
| | |
| |
\ . \ . \ .
\ . \ .

Step 3: Simplify the different terms:
3 2 2
3 3
2 (1 ...) 1 ...
2 2
x x x x

| |
= + + + + + +
|
\ .

Step 4: Multiply all the terms together (only until x
2
in this case, thus reject any
term that contains x
3
and above)
An initiative by Raffles Junior College Mathematics Society 2008 RJC Maths Society
Preliminary Examination Strategies and Tips (PEST) Notes 2008 PAGE 32
3 2 2
2
2 2
3 3
2 (1 ...) 1 ...
2 2
1 3 3
1 1 1
8 2 2
1 5 5
(up till term in )
8 16 16
x x x x
x x
x x x

| |
= + + + + + +
|
\ .
| | | | | |
= + + + +
| | |
|
\ . \ . \ .
= + +


Step 5: Use the graphing calculator to
substitute in a value of x within range of
validity to check what you have expanded is
about the same as the original.

o Find range of values of x for which sequence is valid
As mentioned previously, after simplifying to (1 ( ))
n
f x + for some n that is not a positive
integer, the range of validity of the expansion is ( ) 1 f x < where ( ) f x represents some
function involving the variable x.

Example 10.4: Find the range of validity of the expansion of
2 4 1
(2 5 ) (6 5) (1 3 ) x x x

+ + .

Solution: For range of validity, we only need to consider
2
(2 5 ) x

+ and
1
(1 3 ) x

since the
expansion of the term
4
(6 5) x + , with positive integer index, is always valid. Hence we get
5
2
1
x
<

and 3 3 1 x x = < . Combining both conditions, we have
1
3
x < .

10.3. Approximation Using Binomial Expansion

As questions will often tell you to expand the binomial expansion up to a certain r
th
term,
they will later ask you to use this expansion to approximate certain values. Good
manipulation skills are needed to convert the expansion into the required form.

Example 10.5: By putting
2
3
x = into
2
9 3
6 216
x x
x ~ , find the approximate value of
3 in the form
p
q
, where p and q are integers expressed in their lowest terms.

Solution:
Step 1: Substitute in a suitable value of x within range of validity:
2 2 4
9 3
3 18 1944
=
Step 2: Simplify the equation such that the required value is present.
3(1944) 2(108) 4 25 1 5612 3 5612 5612 3 1403
3
3 1944 9720 3 9720 9720 810
3

= = = = =
or alternatively,
1 5612 9720 2430
3
9720 5612 1403
3
= = =

Note that two different answers can be gotten and will be accepted during examinations.
The slight discrepancy arises due to the factor of 3 which changes the approximated value.
Original
Terms
1 x
2
x
1 1 x
2
x
3
2
x
3
2
x
2
3
2
x
3
3
2
x
2
3
2
x
2
3
2
x
3
3
2
x
4
3
2
x
Figure 10.1: Multiplication Table
When Performing Expansion

An initiative by Raffles Junior College Mathematics Society 2008 RJC Maths Society
Preliminary Examination Strategies and Tips (PEST) Notes 2008 PAGE 33



PURE
MATHEMATICS

CHAPTER 11
POWER SERIES

In a continuation to the previous topic, the concepts involved in power series are similar. In
this chapter, we look at how to convert functions into polynomial form using the Maclaurins
Theorem as well as perform approximation. A good grasp on differentiation is essential here.

11.1. Basic Definitions
We can express a function ( ) P x in the form:

2 3
0 1 2 3
( ) ... ...
n
n
P x a a x a x a x a x = + + + + + +

We can expand many functions into an infinite series like the one above, known as a power
series, to facilitate problem solving. (You should already have an idea of what a series is
from Chapter 8)

The Maclaurins Theorem is one such method which aims to convert a function ( ) P x into
polynomial form, with respect to (0) P . The formula for Maclaurins Series is given as:

( )
2
0
(0) '(0) "(0) (0)
( ) ...
0! 1! 2! !
k
k
k
f f f f
f x x x x
k

=
= + + + =



Here are a few things that you need to know before you begin:
The coefficient,
n
a , in front of
n
x , is:

( ) th
(0) The derivative of the function evaluated at 0
or
! !
n
f n x
n n
=


Some common expansions can be found in MF15, so use them whenever possible!
Also note that you may substitute the x in those equations with something else such
as kx (e.g. 2x, 5x), so that you obtain a required expansion without having to
differentiate.
Some questions require you to substitute one power series into another, for example
ln(1 cos ) x +

The standard question for Maclaurins Series requires you to:
Obtain the first few orders of derivatives for the given equation. How many you need
depends on how many terms the question wants. Implicit differentiation is useful
here, especially for the more complicated functions.
Evaluate the derivatives at 0 x = . That is, find
( )
(0)
k
f for the derivatives you just
obtained. Remember, your main goal is to find the coefficient a
n
.
Plug the values you just obtained into the formula for Maclaurins Series, simplify,
and you have obtained your Maclaurins Series!

An initiative by Raffles Junior College Mathematics Society 2008 RJC Maths Society
Preliminary Examination Strategies and Tips (PEST) Notes 2008 PAGE 34

GC TIP
To confirm your answer, you may use the GC function nDeriv(f(x),x,0) to
find (0) f ' and compare with the value that you found. However it is important to
note that the graphing calculators accuracy for the derivative at that point is up to
5 decimal places.


GC TIP
After obtaining the Maclaurins series, graph what you obtained and the original
power series to see whether the shape of the former is the same and is close to the
original function


11.2. Solving Questions with Maclaurins Series

After obtaining the Maclaurins Series, the questions can ask you to do a few things:
Verify the expansion you obtained using a standard expansion in MF15 (see Tutorial
Q5, Q7) or a given expansion (see Tutorial Q8):
o Expand the terms in the given expression and then simplify to get the same
expansion from Maclaurins Series. If you dont see the suggested standard
expansions in the original equation, you may need to manipulate it such that you
do.
o If the expansion is such that a term is contained within another (see Tutorial
Q5), first expand the term outside, treating the term inside as x. Then expand
the term inside so that the expansion of the inside term is contained within the
expansion of the outside term.
o Sometimes what needs to be expanded is not given (see Tutorial Q7). In this
case, look for the non-polynomial terms that have standard expansions in MF15
(like
x
e ) and expand those.
o Getting something different from your Maclaurins Series? There is probably a
mistake in your verification, or your original expansion (more likely), or both.
Please check out by using the graphing calculator to graph what you got and also
the original expression!
Find the tangent line of the curve of the Maclaurins Series when 0 x = .
o The equation of the tangent line is just (0) (0) y f f x ' = + .
Perform an approximation when x is small enough for powers of x
r
and above to be
neglected.
o Two main categories:
a. Using x =, show that LHS = RHS (see Tutorial Q6)
i. Deals with a specific case
ii. Substitute x into both the original expression and the expansion and
equate the two
iii. Ignore any terms above x
n
and then try to obtain whatever the question
wants from here.
b. Derive an expression that approximates the original (see Lecture Notes E.g. 4)
i. More commonly seen
ii. Do not immediately remove all terms above x
n
. It is usually not that
simple.
An initiative by Raffles Junior College Mathematics Society 2008 RJC Maths Society
Preliminary Examination Strategies and Tips (PEST) Notes 2008 PAGE 35
iii. If there are non-polynomial terms in the approximate expression, for
example
x
e , sinx , make use of the standard expansion in MF15 for
those terms. You need to modify your Maclaurins Series so that it is
expressed as a standard expansion plus whatever leftovers there may be.
Example 4 of the lecture notes is one such question.
iv. If there is something like (1 )
n
x + in the approximate equation, you may
need to expand that term by Binomial expansion. The concept is similar
to that for the non-polynomial terms.
v. In general, if there are expandable terms in the approximate expression,
perform those expansions in your Maclaurins Series first, and only then
ignore the terms above x
n
in the leftover part of your Series.
vi. For both cases, if you cannot obtain the required equation, it is
probably a good idea to check the Maclaurins Series you obtained for
errors.
c. Find an integral of the original expression. (see Tutorial Q6, Lecture Notes
E.g. 6)
i. Your expansion is a polynomial, so it should be easy to integrate
ii. Fixed integrals can be integrated with the graphing calculator.

11.3. Small Angle Approximation

Performing Maclaurins expansion on the basic trigonometric functions and neglecting terms
beyond
2
x , we have the following expressions
2
sin
cos 1
2
tan
x x
x
x
x x
~
~
~

where the angle x is measured in radians. The approximation is valid when x is small (as a
guide, 0.105 rad x < or 6 x <

)

Questions on small angle approximation are highly variable, so there is no general rule,
except that you should be familiar with the above estimations and the usual trigonometric
relations. Other than using the MF15, here are a few tips:
Substitute out the sin, cos, and tan terms you see in expressions with the
approximations and try to solve the question from there.
Show questions are more common here, so always keep the final expression in mind
while manipulating your expressions and check for errors when you get something
different from what is given.





An initiative by Raffles Junior College Mathematics Society 2008 RJC Maths Society
Preliminary Examination Strategies and Tips (PEST) Notes 2008 PAGE 36



PURE
MATHEMATICS

CHAPTER 12
VECTORS

A relatively new topic for many people, the topic of vectors is essential in the field of physics.
It is understandably difficult to pick it up given the combination of various planes, vectors
and lines that can be hard to visualize at times. However, with practice, confidence, and novel
ways at looking at vectors, this chapter will prove to be enjoyable and fun.

12.1. Interpreting Vectors
Good mastery of this chapter requires you to understand each of the three following
conceptualizations of vectors, and to be able to switch between them with ease, as required
by the question.

As geometrical objects As algebraic objects As linear equations
Vectors
are:
Arrows with direction
and length in a 2- or 3-
dimensional space

Column matrices
e.g.,
2
3
4
| |
|
|
|
\ .
,
1
2
0
| |
|
|
|
\ .

Linear equations
e.g.ax by cz d + + = ,
ex fy gz h + + =

Properties Vectors follow usual
geometrical
principles, e.g. two
vectors can be
parallel,
perpendicular, or can
intersect at an angle.
Geometrical rules
like Pythagoras
Theorem can be
applied.
Can be used to
indicate direction or
position
Vectors can be
related through
equations and can be
manipulated in a
manner similar to
normal algebraic
variables.
Addition,
subtraction and
scalar multiplication
rules apply, but
multiplication and
division rules do not
apply.
Vectors can be
written as a linear
equation by
considering each
dimension
separately.
Solving systems of
linear equations
(particularly
Cartesian equations)
can give us
particular points or
vectors that satisfy
the given conditions.
How they
relate
Geometrical Representation to Column matrix: Magnitudes of the value along
different axis/directions in geometrical representation make up the entries in
the corresponding column matrices.
Column matrix to Linear equation: With the use of variables, these matrices
can be related to one another to give equations, one in each dimension.
An initiative by Raffles Junior College Mathematics Society 2008 RJC Maths Society
Preliminary Examination Strategies and Tips (PEST) Notes 2008 PAGE 37
12.2. Getting Started on Vector Questions

As vectors can get quite complex in geometrical representation after some time, it is
important to take note of some tips that can help you to visualize the problem better:
Step 1: Draw a Diagram. Clear, labelled diagrams help you understand questions
better and avoid careless mistakes. A good way to denote planes will be to use
trapeziums instead of parallelograms, such that its boundaries and 3D perspective can
be seen (refer to Figure 12.1 below). Also, since your paper is 2D and it is hard to
show that 2 lines cross without intersecting, try using wedges (such as those used in
organic chemistry) to denote the line is coming out of the paper to you.


Figure 12.1: Perspective drawing of vectors

Step 2: Have a Plan. Dont panic! Questions often overload you with information that
may not all be necessary for every part. Clarify what you need to prove/find, think of
the steps you need to get there, and follow them systematically.
Step 3: Take note of notations. Dont forget the squiggly lines below (a

), arrows on
top ( AB

), and modulus signs on the sides. (|a|)



In addition, do not forget the following basic concepts as well:
o Equations of Lines and Planes
Vector Equation Cartesian Equation
Equation
of a line
, = + e r a b
Make the subject for the
equations in each of the
three dimensions, then
equate these to get the
Cartesian form
1 2 3
1 2 3
x a y a z a
b b b

= =
Use
1
2
3
a
a
a
| |
|
=
|
|
\ .
a and
1
2
3
b
b
b
| |
|
=
|
|
\ .
b to get vector form
Equation
of a
plane
Vector Cartesian Scalar Product
; , = + + e r a b c
Find a normal to the
plane, n, using b c then
dot-product with a to get
the scalar product form
p = = r n a n
1 2 3
n x n y n z p + + =
Set special conditions, e.g. x y =
0 = , to find three points , , A P Q
on the plane. Using these three
points, we can get the vector
form OA AP AQ = + + r


p = = r n a n
Expand the dot-
product to get
the Cartesian
equation.
2 intersecting planes 2 intersecting lines 2 non-intersecting lines 2 non-intersecting planes
An initiative by Raffles Junior College Mathematics Society 2008 RJC Maths Society
Preliminary Examination Strategies and Tips (PEST) Notes 2008 PAGE 38
o Dot-Product versus Cross-Product
Dot-Product ( a b) Cross-Product ( a b)
Output Scalar Vector perpendicular to both a and b
Formula cosu = a b a b , where u is the
angle between a and b
Magnitude: sinu a b , where u is the
angle between a and b
Direction: Right-Hand Grip Rule
Method of
Calculation
1 1
2 2 1 1 2 2 3 3
3 3
a b
a b a b a b a b
a b
| | | |
| |
= + +
| |
| |
\ . \ .

1 1
2 2
3 3
a b
a b
a b
| | | |

| |

| |
| |

\ . \ .

1 1 2 3 3 2
2 2 1 3 3 1
3 3 1 2 2 1
( )
a b a b a b
a b a b a b
a b a b a b
| | | | | |

| | |
=
| | |
| | |

\ . \ . \ .

Pattern for 23 32
subscript: 13 31
12 21
Used for: Calculating length of projection or
perpendicular distance from point
to plane
Writing equation of a plane
Finding angles between lines
and/or planes
Determining if two lines are
perpendicular
Calculating perpendicular distance
from point to line
Calculating area of triangle /
parallelogram
Constructing a vector
perpendicular to two vectors

o When to Use Dot-Product and Cross-Product

Every now and then, vectors might appear in a shape such as triangle ABC. It is made up of
a, b (vectors denoting lines) and c (vector corresponding to segment AC).

Given that you know the angle BAC,
Length AB = length of projection of AC onto a = cos AC u =
^
| | a c
Length BC = perpendicular distance of AC onto a = sin AC u =
^
| | a c
Tip: Knowledge of trigonometric ratios helps.


~
a
A
B C
~
c
~
b
Figure 12.2: Using trigonometry to develop vector equations
A
B C
u
An initiative by Raffles Junior College Mathematics Society 2008 RJC Maths Society
Preliminary Examination Strategies and Tips (PEST) Notes 2008 PAGE 39
12.3. Tackling Questions on Vectors

Below denote the case-by-case techniques on how to solve questions involving vectors:
To prove/disprove that: Use these strategies:
A point
with position
vector q lies
on a:
line ( = + r a b) Set up and solve (for ) the system of three linear equations
given by: = + q a b
If the system has no solution, the point does not lie on the
line.
plane with
equation
p = r n
Calculate q n . If it is equal to p, the point lies on the plane.
Otherwise, the point does not lie on the plane.
plane with
equation
= + + r a b c
Set up and solve (for , ) the system of three linear
equations given by: = + + q a b c
If the system has no solution, the point does not lie on the
plane.
plane with
equation
ax by cz d + + =
If
1 2 3
q q q = + + q i j k, calculate
1 2 3
aq bq cq + + . If it is equal
to d, q lies on the plane. Otherwise, q does not lie on the
plane.
Two vectors
a and b are:
parallel Set up and solve (for ) the system of linear equations given
by = a b.
If it is obvious that b is not a multiple of a, a and b are not
parallel.
Or calculate = a b. If it is 0, a and b are parallel.
perpendicular Calculate a b. If it is 0, a is perpendicular to b.
Otherwise, a is not perpendicular to b.
In 3
dimensions,
two lines
m: r = a +
b and
n: r = c +
d:
are parallel Show that their direction vectors (b and d) are parallel (see
above).
intersect at a
point
Show that their direction vectors (b and d) are not parallel
(see above).
Set up and solve (for , ) the system of linear equations
+ = + a b c d
If the system has a unique solution, the lines intersect at
exactly one point.
Obtain its position vector by substituting the solution for
into m or that for into n.
are skew lines Show that their direction vectors (b and d) are not parallel
(see above).
Set up and solve (for , ) the system of linear equations
+ = + a b c d
If it has no solution, the lines are skew lines.

An initiative by Raffles Junior College Mathematics Society 2008 RJC Maths Society
Preliminary Examination Strategies and Tips (PEST) Notes 2008 PAGE 40
To prove/disprove that: Use these strategies:
A line with
equation
= + r a b
lies on a plane with
equation p = r n
By substitution and direct calculation, show that 0 = b n .
Then, again by substitution and direct calculation, show that
p = a n
is parallel to a
plane with equation
p = r n
By direct calculation, show that 0 = b n .
Then, by direct calculation, show that p = a n .
intersects a plane
with equation
p = r n
By substitution and direct calculation, show that 0 = b n
Set up and solve (for ) the equation ( ) p + = a b n
Substitute the solution into + a b to get the position
vector of the intersection point.
is perpendicular to
a plane with
equation r n = p
Show that b is parallel to n (see above).
Two planes
1
: t r m
p =
and

2
: t r n
p =
are parallel Show that m and n are parallel (see above).
intersect along a
line
Show that m and n are not parallel (see above).
To find the equation of the line, expand the dot product
equation for both planes to get
1
:
1 1 2 3
: mx m y m z p t + + =
and
2 1 2 3
: n x n y n z q t + + = .
Consider the two equations as a system of equations.
Then, solve for x and y in terms of z. If the solution is
x a bz = + and y c dz = + ,then the equation of the line is:
0 1
a c
b d
| | | |
| |
+
| |
| |
\ . \ .

Or set 0 x =

and solve the system to get a point X on the
line.
Then set 0 y =

and solve the system to get a point Y on the
line.
Write the equation of the line in the form XY + x

where x
is the position vector of X.
Three
planes
do not intersect First, take the Cartesian equations of the 3 planes as a
system of 3 linear equations in x, y and z.
If the system has no solution, the planes do not intersect.
If the system has a unique solution
0
x ,
0
y and
0
z , the planes
intersect at 1 point with position vector
0 0 0
x y z + + i j k .
If x and y can be solved in terms of z, the planes intersect along
a line.

intersect at a
point
intersect along a
line


An initiative by Raffles Junior College Mathematics Society 2008 RJC Maths Society
Preliminary Examination Strategies and Tips (PEST) Notes 2008 PAGE 41
Below denote the strategies on tackling finding-type questions:
Finding an angle Finding a vector Finding a length
AB
Finding an equation
Problem Find the angle
between two lines,
a line and a plane,
or two planes
Given certain
conditions, find a free
vector or the position
vector of a point
Find the distance
between two points
A and B
Find the equation of a
line or a plane
satisfying certain
conditions
General
Idea
Calculate dot
product and take
cos
-1
or sin
-1
of this
value.
Solve simultaneous
equations or use
cross-product / ratio
theorem
Use length of
projection or
calculate the
modulus of a vector
Find enough
information to define
the line / plane
Specific
strategies
Step 1: For a line,
use the direction
vector. For a
plane, use a
normal to the
plane

Step 2: Draw a
diagram to
determine which
angles are
involved

Step 3: Find the
dot-product of the
vectors
subtending the
angle. If you want
the acute angle,
take the absolute
value of dot
product

Step 4: If finding
the angle between
a line and a plane,
take sin
-1
of the
dot product. If
not, take cos
-1
of
the dot product.
Case 1: Finding a
vector perpendicular
to two given vectors
Use the cross-product
Case 1: Position
vectors of A and B
are known
AB OB OA =


Case 1: For a line
Equation: = + r a b


Method 1: Find a
direction vector and a
point on the line

Method 2: Get 2
points A and B on the
line to get direction
vector AB


Case 2: Ratio of
lengths given
Use ratio theorem
Case 2: Only the
position vector of A
is known
Method 1: Find the
position vector of B
(see left, Finding a
vector) and
calculate AB



Method 2: Express
AB

as the
projection of AC


onto a given line
where C is a point
with a known
position vector

Method 3:
Sometimes, other
geometrical
identities may need
to be used, e.g. sine
rule, cosine rule,
Pythagoras
theorem, etc.
Case 3: Finding the
position vector of a
point: Think of the
point as the
intersection between
lines and/or planes,
then relate the
equations of the
relevant lines/planes
and solve the system
of linear equations
Case 2: For a plane
Equation: (1)
p = r n , or (2)
= + + r a b c


Step 1: Get 2 vectors
parallel to the plane.
Use these directly in
(2) or use their cross-
product as a normal
to the plane in (1)

Step 2: Find the
position vector of a
point on the plane
and use this to
calculate p in (1) or
use it directly as a in
(2).
An initiative by Raffles Junior College Mathematics Society 2008 RJC Maths Society
Preliminary Examination Strategies and Tips (PEST) Notes 2008 PAGE 42



PURE
MATHEMATICS

CHAPTER 13
COMPLEX NUMBERS

This is a relatively straightforward topic. For the first part you will be required to manipulate
complex numbers in its various forms, solve simultaneous equations and calculate the roots of
a polynomial. In the second part, you will be required to solve problems involving loci in the
complex plane.

13.1. Basic Definitions

The following diagram illustrates the argument arg( ) c , modulus c and conjugate * c of a
complex number c a bi = + . Do take note of the conditions imposed where needed.


Figure 13.1: Representation of a complex number in the Argand plane

13.2. Forms of a Complex Number

Here are the other basic forms of a complex number c a bi = + in its Cartesian form. When
in doubt, draw and denote the complex number on the Argand plane to find out the relevant
r and values.
Trigonometric Form: (cos sin ) r i u u +
o To convert from Cartesian form to trigonometric form, use the formulae
2 2
r a b = + and
1
tan
b
a
u

| |
=
|
\ .
. As the value of
2 2
( , )
t t
u e

from the graphing
calculator, do check on the quadrant that the complex number lies in to find the
correct value of u .
o To convert from trigonometric form to Cartesian form, use cos a r u = and
sin b r u =
Exponential Form:
i
re
u
where r is the modulus of the complex number and u is the
argument.
o To convert from Cartesian form to exponential form, utilize the above method to
find r and , then express it in the form
i
re
u
.
o To convert from exponential form to Cartesian form, use cos a r u = and
sin b r u =
Hint: Remember that 0 r > and ( , ]. u t t e
c a bi = +

arg( ) c u =

0 Re
* c a bi =
2 2
c a b = +

Conditions:
When 0 c = , arg( ) c
is undefined
arg( ) ( , ] c t t e

arg( *) c u =

Im
An initiative by Raffles Junior College Mathematics Society 2008 RJC Maths Society
Preliminary Examination Strategies and Tips (PEST) Notes 2008 PAGE 43
13.3. Basic Operations

You need to be well-acquainted with the various operations that can be performed using
complex numbers. Please refer to your notes for examples.

o Solving Simultaneous Equations
Similar to that in real numbers, solving simultaneous equations involving complex numbers
involves the same procedures as you will do if the unknowns are real. There is no need to
panic. There are two methods:
i. Direct elimination of one unknown through manipulation of equations
ii. Expressing one unknown in terms of the other

Example 13.1: Solve for the complex numbers z and w in the following equations:
(1 ) (2 ) 3 4 (1)

(3 ) 1 5 (2)
i z i w i
iz i w i
+ + = +
+ + = +


Solution A: Using method (i):
Using the idea of complex conjugates which will help convert complex numbers to real
numbers, multiply equation (1) by a factor of 1 i . Then multiply equation (2) by 2i (the
choice of complex number used is such so as to eliminate unknown z thereafter):
2 (1 3 ) 7 (3)

2 ( 2 6 ) 10 2 (4)
z i w i
z i w i
+ = +
+ + =


Adding equations (3) and (4) together, we have ( 1 3 ) 3 i w i + = which gives
3
1 3
i
w i
i

= =
+
. Substituting w i = into equation (2): (3 ) 3 1 iz i i iz i + + = + =
1 5 2 2 i iz i z + = = .


TIP
In the first step, always find the unknown which is easier to eliminate. In this case,
since the coefficient of z in equation (2) is i, thus it is more flexible to be
manipulated either by multiplying by a suitable real number or a multiple of i to
convert the coefficient of z to a real number.

Note: You can multiply equation (2) by a factor of 2i . Try it to see whether you get the
same answer.

Solution B: Using method (ii):
Rearranging equation (2), we have
1 5 (3 )
(*)
i i w
z
i
+ +
= . Substitute into equation (1)
to get
1 5 (3 )
(1 ) (2 ) (1 )( 1 5 (3 ) ) (2 ) 3 4
i i w
i i w i i i w i w i
i
+ +
+ + = + + + = + .
Simplifying and removing common terms on both sides of equation, we have
4 6 ( 4 2 ) (2 ) i i w i w + + + + = 3 4 ( 2 ) 1 2 i i w i w i + + = = . Then substituting into
equation (*) to get 2. z =

An initiative by Raffles Junior College Mathematics Society 2008 RJC Maths Society
Preliminary Examination Strategies and Tips (PEST) Notes 2008 PAGE 44

TIP
Use the easier one of the original two equations to perform the substitution so that
it does not become too complicated upon solving the other equation.


GC TIP
Use your graphing calculator to perform your basic operations of complex numbers!
Remember little marks if any are given for manipulation (e.g. addition,
multiplication etc.) and doing it manually usually results in many computational
errors. For example, it is easy to forget that
2
ai a = and
2 2
( )( ) a bi a bi a b + = +
for any real numbers a and b. In addition, use the GC to check whether the answers
gotten for the unknowns are correct.

13.4. Geometrical Effect of Manipulating Complex Numbers

o Addition and Subtraction
Complex numbers, when represented in the Argand diagram, are like vectors. The complex
number c represented in the Cartesian form c a bi = + tells us that we have to move a units
in the positive real axis and b units in the positive imaginary axis to locate c. Just like
vectors, adding and subtracting complex numbers are done by resolving the imaginary and
real parts of all the complex numbers coordinates.

Remember when performing such manipulations, always convert the complex numbers to
Cartesian form (see part 2) to simplify matters. If necessary, convert it back to trigonometric
or exponential form as seen in part 2.



Figure 13.2: Representation of addition and subtraction of complex numbers

o Multiplication and Division
In exponential form, multiplication and division of complex numbers represent a
transformation of the resultant complex number a certain angle about the origin and also its
length. Remember to convert complex numbers to exponential form before manipulation.
This is also important in your manipulation of complex numbers in geometry in the complex
plane.

Let
1
1 1
i
c re
u
= and
2
2 2
i
c r e
u
= be two complex numbers. Then:
Re
Im
3
1 3
5 6
3 2
a i
b i
c i
a b c i
= +
= +
= +
+ =
a
b
-c
0
a
b
c
Re
Im
0
An initiative by Raffles Junior College Mathematics Society 2008 RJC Maths Society
Preliminary Examination Strategies and Tips (PEST) Notes 2008 PAGE 45

1 2
( )
1 2 1 2
.
i
c c r r e
u u +
= This is represented by turning anticlockwise by an angle
2
u radians
from
1
c and the extension of the length of
1
c (which is
1
r ) by a multiple of
2
. r

1 2
( )
1 1
2 2
.
i
c r
e
c r
u u
= This is represented by turning clockwise by an angle
2
u radians from
1
c and the contraction of the length of
1
c (which is
1
r ) by a multiple of
2
. r

For polygons on the Argand diagram, make use of your knowledge in vectors and geometry
to solve problem. For example, if points , , , A B C D form a square in clockwise order and
AB a bi = +

, then DC a bi = +

. Also,
a bi
AD BC b ai
i
+
= = =

(note it is a
2
t
clockwise
turn of vector AB

and AB AD DC BC = = =

.

13.5. Solving Polynomial Equations with Real Coefficients

You should be familiar with the Remainder and Factor Theorem, which states that for a
polynomial ( ) P x , if ( ) 0 P x o = for some o , then o is a root of the equation ( ) 0 P x = and
( ) ( ) ( ) P x x Q x o = where ( ) Q x is another polynomial. (Note x o is a factor of ( ) P x ).

Before finding all the roots, always find the real roots first before going out to find the
complex roots. In that case you simplify the factorization process. Furthermore, always
remember that complex roots (excluding real roots) occur in conjugate pairs so you should
have an even number of complex roots.

Example 13.2: Find all the roots of
4 3 2
2 2 2 3 0 x x x x = and factorize ( ) P x =
4 3 2
2 2 2 3 x x x x as a product of quadratic and linear factors.

Solution: Since ( 1) (3) 0 P P = = , by Remainder and Factor Theorem, 1 x + and 3 x are
factors of ( ). P x Using long division,
2
( ) ( 1)( 3)( 1) P x x x x = + + . To find the remaining two
roots, we solve
2
1 0 x + = which gives x i = . Thus the four roots of ( ) 0 P x = are
1, 3, x i = .


TIP
If a bi + is a root of ( ) P x , then a bi is also a root of ( ) P x . (Important Fact)
Then a quadratic factor of ( ) P x is represented by ( ( ))( ( )) x a bi x a bi + =
2 2 2
2 ( ) x ax a b + + . You can work it out by direct cross multiplication even if you
do not remember it.

13.6. Loci involving Complex Numbers

In this section, you need to be acquainted with how a locus is determined either (i) by the
distance from a point in the Argand plane, (ii) distance between two points, and (iii) the
angle subtended from a point and the positive real axis.

Recall that, if z is a complex number:
Given ( ) z a bi r + = , then the locus represents a circle with centre ( , ) a b in the
Argand plane with radius r
An initiative by Raffles Junior College Mathematics Society 2008 RJC Maths Society
Preliminary Examination Strategies and Tips (PEST) Notes 2008 PAGE 46
Given
1 1 2 2
( ) ( ) z a b i z a b i + = + , then the locus represents a line passing through
the point
1 2 1 2
,
2 2
a a b b | | + +
|
|
\ .
in the Argand plane and is perpendicular to the line
connecting the points
1 1
( , ) a b and
2 2
( , ) a b .
o In the case of an inequality, say
1 1 2 2
( ) ( ) z a b i z a b i + s + or
1 1
( ) z a b i + <
2 2
( ) z a b i + , take one of the two points,
1 1
( , ) a b and
2 2
( , ) a b , and determine
whether it satisfies the inequality. If so, then shade the side separated by the
locus whereby the point chosen lies on, otherwise, shade the opposite side.


Figure 13.3: Finding the right side to shade in the inequality
1 1 2 2
( ) ( ) z a b i z a b i + s +

Given arg( ( )) z a bi u + = , then the locus represents a line that originates from ( , ) a b
in the Argand plane, but not including it, such that the angle subtended
anticlockwise from the positive Re axis to the locus is radians.


EXAM
TIP
You need to describe the loci in the examinations at times, so it is important to
remember the details. For a circle, describe the coordinates its centre in the
Argand plane and its radius.

o Calculating maximum and minimum arg( ( )) z a bi + involving lines and circles
To find the maximum and minimum arg( ( )) z a bi + with respect to a locus ( ) z x yi r + =
from the point ( , ) a b on the Argand plane, draw two lines which are tangent to the locus
defining the circle. Then, use the following tips to find the answer:

Step 1: Use trigonometry to determine the angle
1
u between point ( , ) P x y = , the
point O = ( , ) a b and the point of tangency T:
1
2 2
sin
( ) ( )
r
a x b y
u =
+
. Note that
2
PTO
t
Z = .
Step 2: Use trigonometry to deduce the angle
2
u between point ( , ) P x y = , the origin
( , ) O a b = and the Re axis (that is
2
tan
y b
x a
u

=

)
Step 3: Add or subtract
2
u from
1
u accordingly, or vice versa. (Sketch it so that you
have an idea on which quantity to subtract from which)
1 1
( , ) a b

2 2
( , ) a b

0
Im
Re

1 1 2 2
( ) ( ) z a b i z a b i + s +

Using the point
1 1
( , ) a b for
example, since substituting the
values of this point satisfies the
inequality, we shade this side
containing this point.

An initiative by Raffles Junior College Mathematics Society 2008 RJC Maths Society
Preliminary Examination Strategies and Tips (PEST) Notes 2008 PAGE 47
Step 4: Add in the appropriate minus sign in front if necessary (since an angle
clockwise from the positive Re axis is negative)

Example 13.3: Find the minimum and maximum value of arg( ) z given that z represents all
complex numbers which satisfy (3 4 ) 2. z i + =


Figure 13.4: Representation of the loci (3 4 ) 2. z i + =

and finding of arg( ) z

Solution: We have
1 2 1 2
2
sin 0 sin 0 0 0 0.41152 rad
5
P T P T P T P T Z = Z = Z = Z = (5sf). Now
consider 0 : P R Z
4
tan 0 0 0.92730 rad
3
P R P R Z = Z = (5sf). Thus the minimum value of
arg( ) 0.92730 0.41152 0.516 rad z = = (3sf), and the maximum value of arg( ) z =

0.92730 0.41152 1.34 rad + = (3sf).

o Calculating maximum and minimum ( ) z a bi + involving lines and circles
To find the maximum and minimum ( ) z a bi + with respect to a circle, from the point
( , ) a b on the Argand plane, draw a line connecting the point ( , ) a b through the centre of the
circle.
The minimum ( ) z a bi + is achieved when the line first touches the locus of the
circle.
The maximum ( ) z a bi + is achieved when the line next touches the locus of the
circle.

Example 13.4: Find the minimum and maximum value of ( ) z a bi + given that z represents
all complex numbers which satisfy ( ) . z x yi r + =

Solution: Let P and Q represent the points ( , ) a b and ( , ) x y respectively. Then the distance
between P and Q is
2 2
( ) ( ) x a y b + . Thus
2 2
min ( ) ( ) ( ) z a bi x a y b r + = + while
2 2
max ( ) ( ) ( ) z a bi x a y b r + = + + .



0
(3 4 ) 2 z i + =
R
1
T
2
T
P
2
Im
Re
An initiative by Raffles Junior College Mathematics Society 2008 RJC Maths Society
Preliminary Examination Strategies and Tips (PEST) Notes 2008 PAGE 48




STATISTICS

CHAPTER S1
PROBABILITY

This chapter is probably the most difficult chapter among the six statistics chapters because
all kinds of scenarios can come up in questions, and there is no fixed way to tackle each
question. However, do note that a probability question is often split into different parts and it
is good to know the various tactics needed to be employed as part of the learning criteria.

S1.1. Basic Definitions

Knowing the following definitions and formulae well will help you immensely in the
examinations by enabling you to understand and tackle the question quickly.

An event A is said to be independent if the occurrence of another event B does not
affect the probability of event A occurring. Thus:
( | ) ( ) ( ) ( ) ( ) P A B P A P A B P A P B = =
Two events A and B are said to be mutually exclusive if the probability of both
events occurring simultaneously is zero. Thus:
( ) 0 A B P A B = C =

S1.2. General Tips and Formulae

There are no specific, fixed methods to tackle probability questions since they can come up
in any form. But it is good to note some techniques that can help you tackle such problems:

Use set diagrams to illustrate the problem
(when you are unsure or if it is complicated)
Use the method of complement if possible as
it is especially useful if it is tedious to
compute directly given many cases.
o ( ) 1 ( ) P A P A' =
Method of splitting into cases:
o Be systematic and careful so that you do not double-count or miss out cases
If possible, utilize the Principle of Inclusion and Exclusion:
o For two events A and B, ( ) ( ) ( ) ( ) P A B P A P B P A B = + .
o If A and B are mutually exclusive events, use formula above to solve.
Solving Find probability of A given B questions:
o
( )
( | )
( )
P A B
P A B
P B

=
o If A and B are independent events, use formula above to solve.
o Note that P(B) is in the denominator! This is because ( | ) P A B is essentially the
probability of finding B occurring with A in the reduced sample space of event B
Use your permutations and combinations (P&C) to count cases as it is often faster
and less confusing. It is good to know it since examiners can incorporate such
concepts in probability
A B
A
A'
Figure S1.1: Illustration of set theory
A B
An initiative by Raffles Junior College Mathematics Society 2008 RJC Maths Society
Preliminary Examination Strategies and Tips (PEST) Notes 2008 PAGE 49
o Always choose (perform
n
C
r
) before permuting the positions of the chosen objects
to avoid double-counting cases
Know your definitions well:
o Though it is highly unlikely that you will be asked to regurgitate definitions,
understanding definitions are always the first step to understanding a topic
because they help to form our fundamental concept of the chapter
a. What does random mean?
b. Do you know the difference between sample space, sample point and event?
c. Can you illustrate the concepts of complement, intersection and union on a
set diagram?
Practise, practise, practise

S1.3. Probability Tree

This is useful in cases where there are a few independent events
(usually 2 or 3) with fixed probabilities (e.g. A occurs with
probability , B occurs with probability ) and one of the events
make up the entire sample space (e.g. A in this case)

There are 2 types of questions
Case 1: Finding ( ) P A B , ( ) P A B' , ( ) P A B ' or
( ) P A B ' '
o They are solved by multiplying 2 relevant probabilities
o For example, ( ) ( ) ( ) (1 ) P A B P A P B o | ' ' = =
Case 2: Finding ( ) P B or ( ) P B'
o They are solved by adding up probabilities of branches that lead to required event,
and then applying the method for Case 1.
o For example, ( ) ( ) ( ) P A P A B P A B ' = + , the individual probabilities can then
be solved using the method in Case 1.

S1.4. Solving Choosing Questions

Such questions are those where you have to select n people (or objects) to form a committee
(or collection) out of m people (m n > ) which can be broken down into distinct subgroups
with restrictions on the number of people from each of these subgroups that are selected.
The general rule is that the denominator is always
( )
m m
n n
C =

The numerator would then be
1 2
1 2
1 2
!
! ! !
k
k
k
a a a
n
n n n
n n n
| | | | | |
| | |
| | |
\ . \ . \ .

where
i
a is the number
of people in each of the k subgroups,
1 2 k
a a a m + + + = and
1 2 k
n n n n + + + =
Note the multiplication of
1 2
!
! ! !
k
n
n n n
to permute the positions. These questions usually
combine the above strategy as well as probability rules (like the complement method and
rules of conditional probability).


A
A

1

1

B
B

1
B
B
Figure S1.2: A
probability tree
An initiative by Raffles Junior College Mathematics Society 2008 RJC Maths Society
Preliminary Examination Strategies and Tips (PEST) Notes 2008 PAGE 50




STATISTICS

CHAPTERS S2 & S3
DISTRIBUTION

This topic touches on the various methods to find the probability of an event occurring given
certain conditions, factors and characteristics on how an event occurs. In this compilation of
Chapters S2 and S3, we touch on the methods and tips on the type of distribution method to
use and how to manipulate them.

S2/3.1. Types of Distribution

There are various types of probability distribution. Depending on the conditions, we can use
different types of methods to categorize and analyze this distribution:

S2/3.2. Types of Questions

Many types of distribution questions exist, and depending on the amount of data and
information given, and your knowledge of the basic operations and manipulations, the types
of questions have been categorized here:

o Questions When the Distribution is Given
Usually you are expected to be able to recognize a binomial distribution without the
question stating it. Here are some features of questions on Poisson and Normal distributions.

Example S2/3.1: (Poisson: Problems involving more than 1 variable) A certain species of
plant may be attacked by 2 distinct types of caterpillars, A and B. The no. of types A and B
on the plant are independent and have Poisson distributions with means 2 and 3
respectively. Given that there is at least 1 caterpillar on a randomly chosen plant, calculate
the probability that all the caterpillars on the plant are the same type.



Continuous
Normal
2
( , ) X N o




Discrete
Random Variable
Poisson
Po( ) X
No. of successes
in n trials,
probability of
success p
Occurrence of
an event per
unit time
Measured data,
can take any
value in given
range
Binomial
( , ) X B n p
An initiative by Raffles Junior College Mathematics Society 2008 RJC Maths Society
Preliminary Examination Strategies and Tips (PEST) Notes 2008 PAGE 51
Solution:
Step 1: First, define all variables involved.
Let X be the no. of type A caterpillars on a randomly chosen plant. Then Po(2) X
.

Let Y be the no. of type B caterpillars on a randomly chosen plant. Then Po(3) Y
.

Thus Po(5) X Y +
.

Step 2: Since conditional probability is involved, we need to use the formula ( | ) P A B

( )
( )
P A B
P B

= . Rewrite the question: (i) All caterpillars of the same type: 0 type A, at
least 1 type B or 0 type B, at least 1 type A, (ii) At least 1 caterpillar: At least 1 of
type A or B.
Find
( 0)( 1) ( 1)( 0)
( 1)
X Y X Y
P
P X Y
| | = > + > =
|
+ >
\ .
.
There is at least 1 caterpillar on a randomly chosen plant, calculate the probability
that all the caterpillars. By manipulating the expressions a little further, we can
simply calculate the probability. The answer is 0.173 (3sf).

Example S2/3.2: (Normal: Standard Normal Variable Z) The mass of sugar sold in bags is
normally distributed with mean kg and variance
2
o

kg
2
. It is known that 30% of the bags
of sugar have masses which differ from kg by at least c kg. Find the probability that a
randomly chosen bag of sugar has a mass which differs from kg by less than 2c kg.

Solution:
Step 1: Define variable.
Let X be the mass of a randomly chosen bag of sugar. Then
2
( , ) X N o .
Step 2: State given information. Draw a diagram (see right). Shaded region represents
( ) P x c > . Thus ( ) 0.30 P x c > = .
Step 3: Get rid of the modulus sign and simplify the
information using the symmetry of normal distribution:
( ) 0.15 P x c s =
Step 4: Since we dont know any parameters, use the standard
normal variable
X
Z

o

= :
( ) ( ) 0.15 1.0364
c c c
P Z P Z

o o o

s = s = = (5sf)
Step 5: Restate question in a simple way such that we can use the information we
have found.
To find

( 2 ) ( 2 )
( 2 ) ( 2 2 )
c c
P X c P c X c P Z


o o
| | +
< = < < = < <
|
\ .

2 2
( 2.0728 2.0728) 0.962
c c
P Z P Z
o o
| |
= < < = < < =
|
\ .
(3sf).

o Questions When the Distribution is not Given
For discrete random variables, if the distribution is not given, it is likely to be binomial but
first check that (i) the event results in either success or failure, and (ii) there is a constant
probability of success. Note that is not always the case. For example, if a question gives
details like a machine produces ribbon in which flaws occur randomly at an average rate of
c c +
An initiative by Raffles Junior College Mathematics Society 2008 RJC Maths Society
Preliminary Examination Strategies and Tips (PEST) Notes 2008 PAGE 52
one flaw in 100m, we are given occurrence of an event per unit time. Then use a Poisson
distribution instead.

Example S2/3.3: A clinic maintains a constant stock of 7 bottles of sleeping pills every
month. It is known from past experience the weekly demand for sleeping pills follows a
Poisson distribution with an average of 1 bottle. Find the probability there are exactly 6
months in a year in which no more than 3 bottles are sold in 1 month (1 month = 4 weeks).

Solution:
Step 1: Recognize that there are 2 separate variables here: (i) numbers of bottles sold
in a month and (ii) number of months in a year in which no more than First we
find the probability of no more than 3 bottles being sold:
Let X be the number of bottles sold in a month. Then Po(4) X . Thus probability
of no more than 3 bottles being sold ( 3) 0.43347 P X = s = (5sf).
Step 2: Define a new variable Y: the number of months a year in which no more than
3 bottles are sold. Since the event that no more than 3 bottles are sold in a month
has 2 possible outcomes and constant probability of success, Y follows a binomial
distribution:
Let Y be the number of months in a year in which no more than 3 bottles are sold.
Then (12, 0.43347) Y B . Hence ( 6) 0.203 PY = = (3sf).

o Questions Requiring Use of Formulae in MF15
When we cannot use the graphing calculator to manipulate given information, we have to
use formulae instead provided in MF15.

Example S2/3.4: (Find the least n) In a college with a large population, 58 students in 100
wear contact lenses. Given that out of n randomly selected students the probability of at
least one student wearing contact lenses is greater than 0.99, find the least value of n.

Step 1: Identify parameters: Probability of a student wearing contact lenses, 0.58 p =
Step 2: Identify variable:
Let X be the number of students out of n who wear contact lenses. ( , 0.58) X B n .
Step 3: State given info. ( 1) 0.99 ( 0) 0.01 P X P X > > = <
Step 4: Since we cannot directly use the graphing calculator to find n, p for Binomial
distribution, we need the formula. Rewrite given info so we can manipulate it:

2
lg 0.42
(1 0.58) 0.42 0.01 lg 0.42 lg 0.01 2 5.31
n n
n n

= < < = > =
.
Hence, the
minimum number n is 6. (Note the change of sign because lg0.42 lg1 0 < = , which
makes lg 0.42 negative)

Example S2/3.5: (Others) A machine produces ribbon in which flaws occur randomly at an
average of 1 flaw in 100m. Find the length of ribbon such that the probability that it
contains no flaws is 0.001.

Solution:
Step 1: Define variable
Let X be the numbers of flaws in a ribbon of length (100n) m. Then Po( ) X n .
An initiative by Raffles Junior College Mathematics Society 2008 RJC Maths Society
Preliminary Examination Strategies and Tips (PEST) Notes 2008 PAGE 53
Step 2: Restate question.
Find n such that ( 0) 0.001 P X = = .
Step 3: Evidently we cant use the graphing calculator. We need the formula
!
x
e
x

(MF15) and rewrite question.


Find n such that
0
0.001
0!
n
e n

= .
Step 4: Take both sides to base e and solve for n. ln0.001 6.9078 n = = . Thus length
of ribbon 691m = (3sf).

Example S2/3.6: (More find the least n questions) A doctor wishes to test a new treatment
for a disease. She asks n patients if they are willing to take part in a trial. The probability of
a patient agreeing to take part is 0.3. Find the least n such that the probability that at least
3 patients being willing to take part in the trial is more than 0.8.

Solution:
Step 1: Define variable, restate question.
Let X be the no. of patients out of n willing to take part in the trial. Then
B( , 0.3) X n

. Find n such that ( 3) 0.8 ( 2) 0.2 P X P X > > s < .
Step 2: As it is too tedious to use the formula in MF15 like in the first example, so
use the graphing calculator to plot y=binomcdf(n,0.3,2). Press 2
nd
>TABLE to see
a list of different values y, i.e. ( 2) P X s can take.
From the graphing calculator, when 13, ( 2) 0.20248 n P X = s = , when 14, n =
( 2) 0.16084 P X s = . Thus the least n is 14.

S2/3.3. Approximation to Other Distributions

When performing calculations for a random distribution, it
is possible to approximate it to other distributions to
simplify calculations. The conditions for approximations
are given below:
Binomial to Poisson: Given B( , ) X n p , if
50, 0.1 n p > < such that

5 np < , then Po( ) X np

approximately.
Binomial to Normal: Given B( , ) X n p , if n is
sufficiently large such that 5 np > and (1 ) 5 n p >
, then ( , (1 )) X N np np p

approximately.
Poisson to Normal: Given Po( ) X , and 10 > ,
then ( , ) X N approximately.

For the last two approximations, do not forget to apply
continuity correction.


TIP
When perform approximation to normal distribution, it is easy to forget that one
has found the variance and thus key in the variance into the graphing calculator.
Remember to square root the variance when performing calculations.
Normal
Binomial
Figure S2/3.1: Triangle of
approximations: The possible
approximations among different
types of random distribution
Poisson
= can be approximated to
An initiative by Raffles Junior College Mathematics Society 2008 RJC Maths Society
Preliminary Examination Strategies and Tips (PEST) Notes 2008 PAGE 54
Example S2/3.7: (Binomial to Poisson, Binomial to Normal) When a certain species of seeds
is sowed, 80% will be able to germinate. Of those that germinate, 95% produce pink flowers
and the rest produce white flowers. Using an appropriate approximation, (a) find the
probability that when 100 seeds are sowed the number of seeds that germinate is more than
70 but at most 90.

Solution:
Step 1: Define variable. Since 0.8 p =

is large, this gives us a clue that normal
approximation might be needed. Check np and (1 ) n p .
Let the no. of seeds out of 100 that germinate be X. Since n is large and 80 5 np = >
and (1 ) 20 5 n p = > , (80,16) X N approximately.
Step 2: Calculate answer, dont forget continuity correction.
(70 90) (70.5 89.5) P X P X < < = < < (by continuity correction) = 0.987 (3sf)

(b) At least 4 will germinate and produce white flowers.

Solution:
Step 1: Probability of producing white flowers 0.05 0.8 0.04 = = . Since p is so small
4 5 np = < ! We cannot use normal distribution anymore. Use Poisson instead.
Let Y be the no. of seeds out of 100 that germinate and produce white flowers. Since
4 5 np = < , Y approximates a Poisson distribution. Po(4) Y

approximately.
Step 2: Evaluate the answer; if necessary, use method of complement:
( 4) 1 ( 3) 0.567 PY PY > = s = (3sf)

Example S2/3.8: (Poisson to Normal) A radioactive disintegration gives counts that follow a
Poisson distribution with mean count of 25 per second. Approximating the normal
approximation from the Poisson distribution, find the probability that in 1 second, the count
is between 23 and 27 inclusive.

Solution:
Step 1: Define variable:
Let X be the number of radioactive counts in 1 second. Then Po(25) X .
Step 2: Show why a normal approximation can be used:
Since 25 10 = > , (25, 25) X N

approximately.
Step 3: Calculate answer using continuity correction.
(23 27) (22.5 27.5) P X P X s s = s s (by continuity correction) 0.383 = (3sf)


TIP
Do not mix up the applications of continuity correction and Central Limit
Theorem! (see Chapters S4 & S5 on the Central Limit Theorem) Continuity
correction is done when approximating a random variable defined by a binomial or
a Poisson distribution to a normal distribution. Central Limit Theorem is used
when some n samples chosen from a non-normal distribution, provided 50 n > , can
hence be approximated to a normal distribution.


An initiative by Raffles Junior College Mathematics Society 2008 RJC Maths Society
Preliminary Examination Strategies and Tips (PEST) Notes 2008 PAGE 55




STATISTICS

CHAPTERS S4 & S5
SAMPLING AND TESTING

This topic touches on the various sampling techniques that can be used and the various
associated benefits and limitations. The later part continues on how to define a sample
distribution out a normal distribution. In hypothesis testing, we look at how to make use of
the details in a sample to prove or disprove an alternative hypothesis.

S4/5.1. Sampling Methods

Four types of sampling methods are presented in the A level curriculum. When a question
asks for a choice of sampling method to conduct a sample, take note of a few pointers:
Identify the sampling method used
Explain why sample is biased/unbiased or representative/unrepresentative
o Answer in the context of the question
Suggest a better sampling method if necessary
o Justify answer
o Describe how it is carried out

In addition, given a population mean

and variance
2
o , then for a sample of size n
containing
i
X ,
2
, X N
n
o

| |
|
|
\ .


if the population is normal. If 50 n > , and if the population
is non-normal,
2
, X N
n
o

| |
|
|
\ .
approximately by Central Limit Theorem.

S4/5.2. Hypothesis Testing

When performing hypothesis testing, look out for the following elements in the question:
Statistical Values Key Words
Sample size, n
Sample mean, x
Population mean

0

Whether or not the population is
normally distributed and assumptions
you need to make to carry out the
hypothesis test
Type of test
Outcome of test (accept or reject)

Null hypothesis / alternative hypothesis

1-tail test / 2-tail test

Significance level

Population variance / standard
deviation

An initiative by Raffles Junior College Mathematics Society 2008 RJC Maths Society
Preliminary Examination Strategies and Tips (PEST) Notes 2008 PAGE 56
o Concept of Hypothesis Testing
The population mean
0

is the hypothesized value, and is not the actual mean of the
population. Hypothesis testing is basically testing, if the claim is correct using the
hypothesized population mean, the given / estimated population and variance, and the
sample mean. In a one-tailed test, if ( ) P X x < or ( ) P X x > (see Figure S4/5.1 above) is
smaller or equal to the significance level (shaded area above), then we reject the null
hypothesis and accept the claim (alternative hypothesis). For a two-tailed test, it is the same
as a one-tailed test except that the significance level is halved since there are now two
regions (left and right of the population mean) to consider.

Basically, use t-test when sample is small and variance is unknown, and z-test for all other
situations. (Refer to page 18 of S5 lecture notes)

S4/5.3. Type of Sampling Questions

Generally, the question requires you to determine one of these elements (e.g. the appropriate
significance level, or the outcome of the test) given all the other elements. In the question,
information about the sample may be presented in several ways in the following flow chart:














0

Sig. level

0

x
Figure S4/5.1: One-tailed test: Reject H
0

X X
x
Sig. level
x
Is either population variance or standard deviation given?
Do direct
hypothesis
testing
Is x

or
2
x

given?
Unbiased estimate of
population mean
1
n
x x =



Unbiased estimate of
population variance
( )
2
2
2
1
1
x
s x
n n
| |

|
=
|

|
\ .


Yes
Yes No
Unbiased estimate of population
mean
1
1 2
( ... )
n n
x x x x = + + +


Unbiased estimate of population
variance
2
1
( )
1
s X x
n
=




No, sample date given
An initiative by Raffles Junior College Mathematics Society 2008 RJC Maths Society
Preliminary Examination Strategies and Tips (PEST) Notes 2008 PAGE 57

S4/5.4. Finding the Outcome of a Test

This is the most common form of question. Use the 5-step method for such question:
Step 1: State the null hypothesis H
0
and the alternative hypothesis H
1
(the claim)
Step 2: Write down 1-tail or 2-tail test, and the level of significance % o (given)
Step 3: Decide on the test statistic to be used and determine its distribution.
Step 4: Use the graphing calculator to calculate the p-value.
Step 5: Make a conclusion in the context of the question.

Dont assume sample follows normal distribution unless told so! If 50 n > (large), apply
Central Limit Theorem.

Example S4/5.1: (Find possible sample mean x (not population mean
0
)) Find the largest
possible mean quantity of milk in the 100 cartons such that there is sufficient evidence, at
1% level of significance, that the mean quantity of milk in a carton is less than 1 litre.

Solution: Perform steps 1 3 first. At step 4, using the graphing calculator to calculate the
p-value, p-value ( ) P X a = < (Letting mean quantity of milk be a litres). Since H
0
is rejected,
( ) 0.01 P X a < s . Using the InvNorm function on the graphing calculator to find a, 0.983 a s
For stage 5: Make a conclusion in the context of the question: the least possible mean
quantity of milk is 0.983 litres.

Example S4/5.2: (Finding the significance level which would result in a given outcome) If H
0

is not rejected, find the maximum level of significance of the test.

Solution: Perform steps 1 3 first. For step 4, use the graphing calculator to calculate the p-
value. By referring to the stated outcome of the test, we form an inequality satisfied by the
significance level of the test: using a z-test, p-value = 0.040059. Since H
0
is not rejected,
% 4.01% a < . For step 5, we make a conclusion in the context of the question that the
maximum level of significance is 4.01%. (3sf)

Example S4/5.3: (Find population mean
0
or sample size n) Perform a 2-tail test at 4%
significance level. Given that the null hypothesis is rejected in favour of the alternative
hypothesis, find the range of possible values of
0
.

Solution: Perform Stage 1-3 first. For step 4, use the p-value of ( ) P X x <

or ( ) P X x >

as
given in the question. For 2-tail test, always divide the p-value by two. From question,
( 0.99413) 0.02 P X < s
.
Since
0
or n

is unknown, we convert X to standard normal random
variable Z using the relation
0 0
or
/ /
X X
Z Z
n s n

o

= = to use the invNorm function. Thus

GC TIP
Input sample data into LIST, and perform hypothesis test on this DATA using your
GC to immediately find out population mean and variance. Alternatively, you can
use the GC 1-Var Stats function.
An initiative by Raffles Junior College Mathematics Society 2008 RJC Maths Society
Preliminary Examination Strategies and Tips (PEST) Notes 2008 PAGE 58
0
0.99413
0.02
0.27088 / 80
P Z

| |

|
< s
|
\ .
. Thus from the graphing calculator,
0
0.99413
2.0537
0.27088 / 80

s .
Remembering to give both inequalities if needed (since the square-root has positive and
negative solutions),
0
0.875 s or
0
1.11 > (3sf). Thus, for step 5: making a conclusion in
the context of the question, the range of values is
0
0.875 s or
0
1.11 > .

S4/5.5. Other Questions

Here are some other tips for other questions that can appear in hypothesis testing:
Answering Explain what the significance level means in the context of the question
o General phrasing: It means the probability of concluding that the claim is true
when it is actually not true is (the significance level).
o Phrasing in context: E.g. Manufacturer claims that the mean foot-length is 10cm.
Then a 5% significance level means that the probability of concluding that the
mean foot-length is not 10cm when it is actually 10 cm is 0.05.
Answering State the type of test to be used and the assumptions made questions
o Normally the assumption made is that the population follows a normal
distribution (so as to be able to use z-test or t-test when sample size is small
50 n < )
Answering Compare the outcome of using z-test compared to using t-test questions
(See Tutorial Q10, part (ii))
o Basically the p-value of a t-test is greater or equal to the p-value of a z-test, since
about the population mean, the graph for a t-test is flatter than that of a z-test

o Comparing effect of change in significance level/sample mean/population mean, on
outcome
Based on question, the p-value can be expressed to be of the form ( ) P X x <

or ( ) P X x >

Case 1: p-value
0 0
( ) ( ) or ( )
/ /
x x
P X x P Z P Z
n s n

o

= < = < <
If the adjustment makes the expression on the RHS of Z smaller, p-value decreases. If
not, p-value increases. The outcome is affected accordingly.
Case 2: p-value
0 0
( ) ( ) or ( )
/ /
x x
P X x P Z P Z
n s n

o

= > = > >
If the adjustment makes the expression on the RHS of Z larger, p-value decreases. If
not, p-value increases. The outcome is affected accordingly.









An initiative by Raffles Junior College Mathematics Society 2008 RJC Maths Society
Preliminary Examination Strategies and Tips (PEST) Notes 2008 PAGE 59






STATISTICS

CHAPTER S6
CORRELATION AND
REGRESSION

Basic concepts developed in this chapter include scatter diagrams, Product Moment
Correlation Coefficient and Linear Regression (Least Squares Regression Lines). The most
important techniques are used on graphing calculators. The exact commands and pressing of
buttons will not be included in this page, however.

S6.1. Scatter Diagram

In this chapter, we aim to establish a linear correlation, if any, between two chosen variables
(usually one is independent and the other is dependent). The variables can have:
(Strong) Positive / Negative linear correlation (This usually means 1 r ~ )
Non-linear relation (This usually means 0 r ~ )
No relation (r-value can be hard to determine at times)

Note the differences between no relation and non-linear relation. Two variables can have
a relationship, such as the activity of a radioactive source A and the number of radioactive
atoms present x, but their relationship of x and A may not be linear.

S6.2. Product Moment Correlation Coefficient (r-value)

Calculations for the r-value can be done using graphing calculator, as well as the formulae
provided. However, the latter are not required for A level.

Implications of the r-value:
1 r = : Perfect positive linear correlation
0 r = : No linear correlation
1 r = : Perfect negative linear correlation

Remember that the r-value can give us an idea on the linear correlation between two
variables. Ultimately, it should not be the only factor on such issues since it is possible for a
plot to have a high absolute r-value yet most of the points are scattered.



GC TIP
After finding out the r-value, use your graphing calculator to plot the least squares
regression line of the scatter diagram to check whether it actually gives a good fit of
the points and also to check for outlying points. If there are many outlying points
yet the absolute value of r is high (i.e. close to 1), there is probably no correlation
between the two variables involved.
An initiative by Raffles Junior College Mathematics Society 2008 RJC Maths Society
Preliminary Examination Strategies and Tips (PEST) Notes 2008 PAGE 60
Note that if 0 1 r < < , it cannot be determined exactly in all cases whether there is a
positive or negative linear correlation. Use the graphing calculator to find out.

S6.3. Linear Regression

To find the equation of the least squares regression line, always make it in the form of
y a bx = + where x is the independent variable and y is the dependent variable.

To find out which is the independent variable, either determine by logic which variable will
definitely affect the other or try to find a pattern in the tabulation of one of the variables
(e.g. standard increments of 20). Remember that:
Data with linear relation definitely have a large r-value; but large r-value does not
necessarily indicate linear relationship (see above)
The existence of a linear relationship does not imply causality

o Interpolation and Extrapolation
When doing interpolation and extrapolation using the equation of the least squares
regression line, substitute the relevant values into the equation. Remember the limitations of
extrapolation (such as getting negative values when it is not possible or lower or higher than
expected values with the equation as compared to the data). For relations whereby it cannot
be determined exactly which variable is independent, use the appropriate least squares
regression line to find out the answer. (e.g. to find value of y given x, use y a bx = + even
though you may think that x depends on y)

o Linearization
The aim is to find the largest r-value, out of the proposed models, using the given data set
provided. The best models will result in the largest r-value. Remember as a check plot the
graph to see whether the model gives a best-fit least squares regression line with all the data
points close to it.











An initiative by Raffles Junior College Mathematics Society 2008 RJC Maths Society
Preliminary Examination Strategies and Tips (PEST) Notes 2008 PAGE 61


PEST NOTES
2008



The following people were involved in the writing, editing and designing of this edition of
Preliminary Examination Strategies and Tips notes, and the editors will like to thank them
for taking time off to complete the various chapter sections:

Tan Chong Min (08S03F) (Overall co-coordinator and editor)
Qian Chen (08S03P)
Khoo Seng Teck (08S06A) (Overall co-coordinator and editor)
Lim Jun Ren (08S06F)
Charlotte Deng Ying (08S06H)
Yin Zhaoxin (08S06I)
Tan Juanhe (08S06N)
Chan Wai Hong Ronald (08S06P)
Milashini Nambiar (08S06R)
Chia Jing Ting Charmaine (08S06S)

In addition, the editors will like to thank the following people for their invaluable support,
guidance and/or review for this set of notes:

Mr. Kok Chee Kean and Mrs. Katherine Ng-Goh Chee Ying (teachers-in-charge of
Raffles Junior College Mathematics Society)









DISCLAIMER

For any feedback on the PEST Notes, please feel free to contact the editors at
rjcpestnotes@gmail.com

The editors reserve the right to make any amendment to the notes after the release date. The
notes are only meant as a guide and the use of the information provided hereby are at the
readers risk. While the editors make every effort to provide accurate and complete
information for students revising for their examinations in the PEST notes, if there are any
errors found in this set of notes, the editors will be pleased to make amends at the earliest
possible opportunity. The editors and the contributors will NOT be responsible for any harm,
loss or damage to human life, property or examination grades caused through the use of the
notes.
appreciation

Você também pode gostar